Final Exam Study Guide

Réussis tes devoirs et examens dès maintenant avec Quizwiz!

The nurse is evaluating the client who is in skeletal traction. When evaluation the pin sites, the nurse would be most concerned with which finding?

Thick, yellow drainage from the pin sites

A patient has had a surgical reduction of an open fracture of the right radius. Which finding should be immediately reported to the HCP?

Temperature of 101.4 F

The nurse is caring for a client who has had a spinal fusion with insertion of hardware. The nurse would be most concerned with which assessment finding?

Temperature of 101.6 F

A client is being discharged to home after application of a plaster leg cast. Which statement indicates the client understands proper care of the cast?

"I need to avoid getting the cast wet

The nurse is giving instructions to a client who sustained a ligament injury who is returning home after a knee arthroscopy. Which statement by the client indicates that the instructions are understood?

"I need to report a fever, redness around my incisions, or persistent drainage to my health care provider."

A chemical explosion occurs at a nearby industrial site. First responders report that victims are being decontaminated at the scene and about 125 works will need medical evaluation and care. The first action of the nurse receiving the report should be:

- Activate the hospital's emergency response plan

The client is brought to the emergency department with partial thickness burns to his face, neck, arms, and chest after trying to put out a car fire. What are the nursing actions for this client?

- Administer oxygen - Assess for airway patency -Evaluate the extremities if no fractures are present Rationale: Prevent hypovolemic shock, maintain airway, initiate IV fluids, and preserve vital organ functioning. The extremities are elevated to assist in preventing shock and decrease fluid from moving into the extremities, especially in the burned upper extremities. Client is kept warm since the loss of skin integrity causes heat loss. Client is placed on NPO due to altered GI function.

The nurse is assessing a client who sustained circumferential burns of both legs. Which assessment would be the initial priority in caring for this client?

- Assessing peripheral pulses

The nurse is caring for a client who had a resection of an abdominal aortic aneurysm yesterday. The client has an intravenous (IV) infusion at a rate of 150 ML/hour, unchanged for the last 10 hours. The client's urine output for the last 3 hours has been 90, 50, and 28 ML (28 ML is most recent). The client's blood urea nitrogen level is 35 mg/DL (12.6 mmol/L) and the serum creatinine level is 1.8 mg/DL (159 mcmol/L), measured this morning. Which nursing action is the priority? a) Check the urine specific gravity. b) Call the health care provider. c) Put the IV line on a pump so that the infusion rate is sure to stay stable. d) Check to see if the client has a blood sample for a serum albumin level drawn.

- Call the health care provider *Following abdominal aortic aneurysm resection or repair, monitor for acute kidney injury. Acute kidney injury can occur because often much blood is lost during the surgery and depending on the aneurysm location they renal arteries may be hypo-perfused for a short period of time.

A nurse gives 1 mg of morphine, but documents that 2 mg of morphine was given. What actions should the nurse take?

- Right click to correct the information within the chart - Obtain a co-signature from another nurse to sign off on the wasted 1mg - Document the correct information and sign your initials and the date - Give a detailed nurse's note to explain the medication error

A client is admitted to the emergency department with chest pain that is consistent with myocardial infarction based on elevated troponin levels. Heart sounds are normal. The nurse should alert the health care provider because these changes are most consistent with which complication? Client's Chart Time: 11:00am 11:15am 11:30am 11:45am Pulse: 92 bpm 96 bpm 104 bpm 118 bpm RR: 24 bpm 26 bpm 28 bpm 32 bpm BP: 140/88 128/82 104/68 88/58 1. Cardiogenic shock 2. Cardiac tamponade 3. Pulmonary embolism 4. Dissecting thoracic aortic aneurysm

- Cardiogenic shock

The nurse is monitoring a client who required a Sengstaken-Blakemore tube because other measures for treating bleeding esophageal varices were unsuccessful. The client complains of severe pain of abrupt onset. Which nursing action is most appropriate? a. Cut the tube. b. Reposition the client. c. Assess the lumens of the tubes. d. Administer the prescribed analgesics.

- Cut the tube Rationale: Spontaneous rupture of the gastric balloon, upward migration of the tube, and occlusion of the airway are possible complications associated with a Sengstaken-Blakemore tube. Esophageal rupture also may occur and is characterized by the abrupt onset of severe pain. In the event of any of these life-threatening emergencies, the tube is cut and removed.

Acetylcysteine (Mucomyst) is prescribed for a client in the hospital emergency department after diagnosis of acetaminophen (Tylenol) overdose. The nurse prepares to administer the medication using which procedure?

- Diluting the medication in cola and administering the patient orally

Which are the most accurate assessment parameters for a nurse to determine adequate tissue perfusion in patient with MODS.

- Level of consciousness, urine output, skin color, temperature

A client who has suffered a large flail chest is experiencing severe pain and dyspnea. The client's central venous pressure (CVP) is rising, and the arterial blood pressure is falling. Which condition should the nurse interpret that the client is experiencing?

- Mediastinal flutter

The client has a radical neck dissection and begins to hemorrhage at the incision site. What immediate action should the nurse take?

- Monitor airway - Monitor vital signs ??? (This was a possible choice option) - Apply manual pressure - Call rapid response team

The nurse manager is discussing the facility protocol in the event of a tornado with the staff. Which instructions should the nurse manager include in the discussion? SATA. 1. Open doors to client rooms. 2. Move beds away from windows. 3. Close window shades and curtains. 4. Place blankets over clients who are confined to bed. 5. Relocate ambulatory clients from the hallways back into their rooms.

- Move beds away from windows - Close window shades and curtains - Place blankets over clients who are confined to bed

The nurse is conducting a session about principles of first aid and discussing the interventions for a snakebite to an extremity. The nurse should inform those attending the session that the first priority intervention in the event of this occurrence is which action?

- Move victim to a safe area away from the snake and encourage victim to rest

The nurse caring for a client with a chest tube turns the client to the side and the chest tube accidentally disconnects from the water seal chamber. Which initial action should the nurse take?

- Place the tube in sterile water

The nurse has given the client instructions about crutch safety. Which statements indicate that the client understands the instructions?

-"I would not use someone else's crutches" - "I need to remove any scatter rugs at home" - "I need to have spare crutches and tips available"

Interventions used for the patient with acute respiratory distress syndrome (ARDS) include:

-Adequate analgesia??? (This is what Prof. H. said on recording) - Aggressive IV fluid resuscitation??? (This was on Taylor's list) - Elevate head of the bed to 30-45 degrees when supine - Monitor hemodynamic parameters and daily weights

In monitoring a client's response to disease modifying antirheumatic drugs, which assessment findings are acceptable responses? (SATA)

-Control symptoms during periods of emotional stress - Normal WBC, platelet, and neutrophil counts - Radiological findings that show no progression of joint degeneration - an increased ROM in affected joints 3 months into therapy

What are the effective interventions to decrease the absorption or increase the elimination of an ingested poison? (SATA)

-Gastric lavage - Activated charcoal - Hemodialysis

A nurse provides teaching to a client who has a newly applied short arm fiberglass cast. Which of the following instructions should the nurse discuss?

-Report numbness or tingling to your provider - Keep arm elevated above the heart during rest - Blow lol air from hair dryer to relieve itching - wrap cast with plastic covering prior to showering

A client is receiving a transfusion of packed red blood cells (PRBCs) and begins to vomit. The BP is now 90/50 from a previous baseline reading of 125/78, and the temperature is now 100.8 F from a previous baseline reading of 99.2 F. The nurse determines that the client may be experiencing which complication? 1.Septicemia 2.Hyperkalemia 3.Circulatory overload 4.Delayed transfusion reaction

1.Septicemia Rationale: There is some type of microorganism from contaminated blood that is causing a negative reaction to the transfusion

Which identifies accurate nursing documentation notation? 1. The client slept through the night. 2. Abdominal wound dressing is dry and intact without drainage 3. the client seemed angry when awakened for vital sign measurement 4. The client appears to become anxious when it is time for respiratory treatment. 5. the client's left lower medial leg wound is 3 cm in length without redness, drainage, or edema.

2. Abdominal wound dressing is dry and intact without drainage 5. the client's left lower medial leg wound is 3 cm in length without redness, drainage, or edema. 1. The client slept through the night.

A client with valvular heart disease who has a clot in the right atrium is receiving a heparin sodium infusion at 1000 units/hr and warfarin sodium 7.5 mg at 5:00 p.m. daily. The morning laboratory results are as follows: activated partial thromboplastin time (aPTT), 32 seconds; international normalized ratio (INR), 1.3. The nurse should take which action based on the client's laboratory results? 1. collaborate with the HCP to discontinue the heparin infusion and administer the warfarin sodium as prescribed 2. collaborate with the HCP to obtain a prescription to increase the heparin infusion and administer the warfarin sodium as prescribed 3. collaborate with the HCP to withhold the warfarin sodium since the cient is receiving a heparin infusion and the aPTT is within the therapeutic range 4. collaborate with the HCP to continue the heparin infusion at the same rate and to discuss use of dabigatran etexilate in place of warfarin sodium

2. collaborate with the HCP to obtain a prescription to increase the heparin infusion and continue the warfarin sodium as prescribed Rationale: when a client is receiving warfarin for clot prevention due to Afib, an INR of 2-3 is appropriate for most clients. Until the INR has achieved a therapeutic range, the client should be maintained on a continuous heparin infusion with the aPTT ranging between 60 and 80 seconds. Therefore, the nurse should collaborate with the HCP to obtain a prescription to increase the heparin infusion and to administer the warfarin as prescribed

Which information should the nurse include when teaching a patient who is newly diagnosed with systemic exertion intolerance disease about self-management?

A gradual increase in daily exercise may help decrease fatigue

The nurse is assigned to care for 4 clients. In planning the client rounds, which client should the nurse assess first?

A patient with asthma who requested a breathing treatment on the previous shift

The nurse is conducting health screening for osteoporosis. Which client is at greatest risk is developing this problem?

A sedentary 65-year-old client who smokes cigarettes

The nurse assumes care of a patient who just returned from surgery for a total laryngectomy and radical neck dissection and notes the following problems. In which order should the nurse address the problems? (Put a comma and a space between each answer choice [A, B, C, D].) a. The patient is in a side-lying position with the head of the bed flat. b. The patient is coughing blood-tinged secretions from the tracheostomy. c. The nasogastric (NG) tube is disconnected from suction and clamped off. d. The wound drain in the neck incision contains 200 mL of bloody drainage.

A, B, D, C Rationale: The patient should first be placed in a semi-Fowler's position to maintain the airway and reduce incisional swelling. The blood-tinged secretions may obstruct the airway, so suctioning is the next appropriate action. Then the wound drain should be drained because the 200 mL of drainage will decrease the amount of suction in the wound drain and could lead to incisional swelling and poor healing. Finally, the NG tube should be reconnected to suction to prevent gastric dilation, nausea, and vomiting.

When anazlysing an EKG rhythm strip of a patient with a regular heart rhythm, the nurse counts 30 small blocks from one R wave to the next. The nurse calculates the patient's heart rate as: 50When preparing to defibrillate a patient. In which order will the nurse perform the following steps? (Put a comma and a space between each answer choice [A, B, C, D, E].) a. Turn the defibrillator on. b. Deliver the electrical charge. c. Select the appropriate energy level. d. Place the paddles on the patient's chest. e. Check the location of other staff and call out "all clear." answer

A, C, D, E, B Rationale: This order will result in rapid defibrillation without endangering hospital staff.

A nurse is caring for a client who had an open abdominal cholecystectomy because of biliary colic. Which nursing action is most important during the postoperative period?

Encourage coughing and deep breathing

A client admitted to the hospital with a suspected diagnosis of acute pancreatitis is being assessed by the nurse. Which assessment findings would be consistent with acute pancreatitis? Select all that apply

A. Abdominal guarding and tenderness B. Left upper quadrant pain with radiation to the back C. Gray-blue color at the flank

A patient who comes to the clinic reports frequent, watery stools for 2 days. Which action should the nurse take first? A. Check the patient's blood pressure B. Draw blood for serum electrolyte levels C. Ask about extremity numbness or tingling D. Obtain the baseline weight

A. Check the patient's blood pressure Rationale: Because the patient's history suggests that fluid volume deficit may be a problem, assessment for adequate circulation is the highest priority. The other actions are also appropriate but are not as essential as determining the patient's perfusion status

After ureterolithotomy, a patient has a left ureteral catheter and a urethral catheter in place. Which action will the nurse include in the plan of care? A. call the HCP if the ureteral catheter output drops suddenly B. clamp the ureteral catheter off when output from the urethral catheter stops C. apply continuous steady tension to the ureteral catheter D. providing teaching about home care for both catheters

A. Call the HCP if the ureteral catheter output drops suddenly Rationale: The HCP should be notified if the ureteral catheter output decreases because obstruction of this catheter may result in an increase in pressure in the renal pelvis. Tension on the ureteral catheter should be avoided to prevent catheter displacement. To avoid pressure in the renal pelvis, the catheter is not clamped. Because the patient is not usually discharged with a ureteral catheter in place, patient teaching about both catheters is not needed.

A patient in the clinic with cystic fibrosis (CF) reports increased sweating and weakness during the summer months. Which action by the nurse would be most appropriate? A. Have the patient add dietary salt to meals B. Teach the patient signs of hypogylcemia C. Suggest decreasing intake of dietary fat and calories D. Teach the patient about pancreatic enzyme replacement

A. Have the patient add dietary salts to meals Rationale: Added dietary salt is indicated whenever sweating is excessive, such as during hot weather, when fever is present, or from intense physical activity. The management of pancreatic insufficiency includes pancreatic enzyme replacement before each meal and snack. This patient is at risk for hyponatremia based on reported symptoms. Adequate intake of fat, calories, protein, and vitamins is important. Fat-soluble vitamins (Vitamins A, D, E, and K) must be supplemented because they are malabsorbed. Use of caloric supplements improves nutritional status. Hyperglycemia caused by pancreatic insufficiency is more likely to occur than hypoglycemia

A patient who is experiencing an asthma attack develops bradycardia and a decrease in wheezing. Which action should the nurse take first? A. Notify the HCP B. Administer IV methylprednisolone (Solu-Medrol) C. Encourage the patient to cough and deep breathe D. Document changes in respiratory status

A. Notify the HCP Rationale: The patient's assessment indicates impending respiratory failure, and the nurse should prepare to assist with intubation and mechanical ventilation after notifying the HCP. IV corticosteroids require several hours before having any effect on respiratory status. The patient will not be able to cough or deep breathe effectively. Documentation is not a priority at this time

A patient admitted with acute kidney injury due to dehydration has oliguria, anemia, and Hyperkalemia. Which prescribed action should the nurse take first? A. Place the patient on a cardiac monitor B. Administer epoetin alfa (Epogen) C. Insert a urinary retention catheter D. Give sodium polystyrene sulfonate (Kayexalate)

A. Place the patient on a cardiac monitor Rationale: Because Hyperkalemia can cause fatal dysrhythmias, the initial action should be to monitor the cardiac rhythm. Kayexalate and Epogen will take time to correct the Hyperkalemia and anemia. The catheter allows monitoring of the urine output but does not correct the cause of the renal failure.

Which assessment finding should the nurse expect when a patient with acute kidney injury (AKI) has an arterial blood pH of 7.30? A. Rapid, deep respirations B. Hot, flushed face and neck C. Persistent skin tenting D. Bounding peripheral pulses

A. Rapid, deep respirations Rationale: Patients with metabolic acidosis caused by AKI may have Kussmal respirations to eliminate carbon dioxide. Bounding pulses and vasodilation are not associated with metabolic acidosis. Because the patient is likely to have fluid retention, poor skin turgor would not be a finding in AKI

The clinic nurse is teaching a patient with acute sinusitis. Which interventions should the nurse plan to include in the teaching session? (Select all that apply) A. Taking a hot shower will increase sinus drainage and decrease pain B. Saline nasal spray can be made at home and used to wash out secretions C. You will be more comfortable if you keep your head in an upright position D. Decongestants can be used to relieve swelling E. Avoid blowing the nose to decrease the nosebleed risk

A. Taking a hot shower will increase sinus drainage and decrease pain B. Saline nasal spray can be made at home and used to wash out secretions C. You will be more comfortable if you keep your head in an upright position D. Decongestants can be used to relieve swelling Rationale: The steam and heat from a shower will help thin secretions and improve drainage. Decongestants can be used to relieve swelling. Patients can use either OTC sterile saline solutions or home-prepared saline solutions to thin and remove secretions. Maintaining an upright posture decreases sinus pressure and the resulting pain. Blowing the nose after a hot shower or using the saline spray is recommended to expel secretions

The nurse observes a student who is listening to a patient's lungs. Which action by the student indicates a need for review respiratory assessment skills? A. The student listens during the inspiratory phase, then moves the stethoscope B. The student instructs the patient to breathe slowly and deeply through the mouth C. The student starts at the apices of the rings, moving down toward the lung bases D. The student compares breath sounds from side to side at each level

A. The student listens during the inspiratory phase, then moves the stethoscope Rationale: Listening only during inspiration indicates the student needs a review of respiratory assessment skills. At each placement of the stethoscope, listen to at least one cycle of inspiration and expiration. During chest auscultation, instruct the patient to breathe slowly and a little deeper than normal through the mouth. Auscultation should proceed from the lung apices to the bases, comparing opposite areas of the chest, unless the patient is in respiratory distress or will tire easily.

A patient is receiving a 3% saline continuous IV infusion for hyponatremia. Which assessment data will require the most rapid response by the nurse? A. There are crackles throughout both lung fields B. The patient's radial pulse is 105 bpm C. The patient's blood pressure increases to 142/94 mmHg D. There is sediment and blood in the patient's urine

A. There are crackles throughout both lung fields Rationale: Crackles throughout both lungs suggest that the patient may be experiencing pulmonary edema, a life-threatening adverse effect of hypotonic solutions. The increased pulse rate and blood pressure and the appearance of the urine should also be reported, but they are not as dangerous as the presence of fluid in the alveoli

The nurse is planning to teach a client with gastroesophageal reflux disease (GERD) about substances to avoid. Which items should the nurse include on this list? Select all that apply

A. chocolate b. coffee c. fried chicken d. peppermint

The nurse is caring for a patient with idiopathic pulmonary arterial hypertension (IPAH). Which assessment information requires the most immediate action by the nurse? A. the epoprostenol (Flolan) infusion is disconnected B. the blood pressure is 98/56 mmHg C. the O2 saturation is 90% D. the international normalized ratio (INR) is prolonged

A. the epoprostenol (flolan) infusion is disconnected Rationale: The half-life of this drug is 6 minutes, so the nurse will need to restart the infusion as soon as possible to prevent rapid clinical deterioration. The other data also indicate a need for ongoing monitoring or intervention, but the priority action is to reconnect the infusion

The nurse is titrating the IV fluid infusion rate immediately after a patient has had kidney transplantation. Which parameter will be most important for the nurse to consider? A. urine output B. creatinine clearance C. heart rate D. blood urea nitrogen (BUN) level

A. urine output Rationale: Fluid volume is replaced based on urine output after transplant because the urine output can be as high as a liter an hour. The other data will be monitored but are not the most important determinants of fluid infusion rate

Prior to administering a client's scheduled dose of enoxaparin sodium, which finding should the nurse evaluate?

Activate partial prothrombin time (aPTT)

Client sustains a contusion to the eyeball after a traumatic injury to the eye with a blunt object

Apply ice to the affected eye

Which information should the nurse include when preparing teaching to a patient who has had an exacerbation of rheumatoid arthritis?

Applying cold packs before exercise may decrease pain

An abdominal cholecystectomy is performed on a client with gangrene of the gallbladder. During the first 24 hours postoperatively, when should analgesics be administered?

As prescribed by a health care provider

Which situation would require the nurse to obtain a focused assessment (select all that apply)? A) A patient denies a current health problem. B) A patient reports a new symptom during rounds .C) A previously identified problem need reassessment. D) A baseline health maintenance examination is required. E) An emergency problem is identified during physical examination.

B) A patient reports a new symptom during rounds .C) A previously identified problem need reassessment. RATIONALE: A focused assessment is an abbreviated assessment used to evaluate the status of previously identified problems and to monitor for signs of new problems. It can be performed when a specific problem is identified or a new symptom is reported.

The nurse is providing discharge teaching for a client with newly diagnosed Crohn's disease about dietary measures to implement during exacerbation episodes. Which statement made by the client indicates a need for further instruction?

B. "I should increase the fiber in my diet"

A student asks the nurse why a peripherally inserted central catheter is needed for a patient receiving parenteral nutrition with 25% dextrose. Which response by the nurse is accurate? a. "The prescribed infusion can be given more rapidly when the patient has a central line." b. "The hypertonic solution will be more rapidly diluted when given through a central line." c. "There is a decreased risk for infection when 25% dextrose is infused through a central line." d. "The required blood glucose monitoring is based on samples obtained from a central line."

B. "the hypertonic solution will be more rapidly diluted when given through a central line." Rationale: the 25% dextrose solution is hypertonic. Shrinkage of RBCs can occur when solutions with dextrose concentrations greater than 10% are administered IV. Blood glucose testing is not more accurate when samples are obtained from a central line. The infection risk is higher with a central catheter than with peripheral IV lines. Hypertonic or concentrated IV solutions are not given rapidly

The nurse is monitoring a client with a diagnosis of peptic ulcer. Which assessment finding would most likely indicate perforation of the ulcer?

B. A rigid, board-like abdomen

After placement of a centrally inserted IV catheter, a patient reports acute chest pain and dyspnea. Which action should the nurse take first? A. Offer reassurance to the patient B. Auscultate the patient's breath sounds C. Notify the HCP D. Give prescribed PRN morphine sulfate IV

B. Auscultate the patient's breath sounds Rationale: The initial action should be to assess the patient further because the history and symptoms are consistent with several possible complications of central line insertion, including embolism and pneumothorax. The other actions may be appropriate, but further assessment of the patient is needed before notifying the HCP, offering reassurance, or administration of morphine

The nurse is caring for a patient who has acute pharyngitis caused by Candida albicans. Which action is appropriate for the nurse to include in the plan of care? A. Teach the patient to sleep in a warm, dry environment B. Teach patient to "swish and swallow" prescribed oral nystatin C. Assess patient for allergies to penicillin antibiotics D. Avoid giving the patient warm food or warm liquids to drink

B. Teach patient to "swish and swallow" prescribed oral nystatin Rationale: oral and pharyngeal fungal infections are treated with nystatin solution. The goal of the "swish and swallow" technique is to expose all the oral mucosa to the anti fungal agent. Warm liquids may be soothing to a sore throat. The patient should be taught to use a cool mist humidifier. There is no need to assess for penicillin allergy because C. albicans infection is treated with antifungals

A 55-yr-old woman admitted for shoulder surgery asks the nurse for a perineal pad, stating that laughing or coughing causes leakage of urine. Which intervention is appropriate to include in the care plan? A. Demonstrate how to perform the Credé maneuver B. Teach the patient how to prefer Kegel exercises C. Assist the patient to the bathroom q3hr D. Place a commode at the patient's bedside

B. Teach the patient how to perform Kegel exercises Rationale: Kegel exercises to strengthen the pelvic floor muscles will help reduce stress incontinence. The Credé maneuver is used to help empty the bladder for patients with overflow incontinence. Placing the commode close to the bedside and assisting the patient to the bathroom are helpful for functional incontinence.

Following a thyroidectomy, a patient reports "a tingling feeling around my mouth." Which assessment should the nurse complete first? A. Verify the serum potassium level B. Test for presence of Chvostek's sign C. Confirm a prescription for thyroid replacement D. Observe for blood on the neck dressing

B. Test for presence of Chvostek's sign Rationale: The patient's symptoms indicate possible hypocalcemia, which can occur secondary to parathyroid injury or removal during thyroidectomy. There is no indication of an urgent need to check the potassium level, the thyroid replacement, or for bleeding

Which assessment finding for an older patient indicated that the nurse should take immediate action? A. dry mucous membranes B. bilateral basilar crackles C. weak cough effort D. barrel-shaped chest

B. bilateral basilar crackles Rationale: Crackles in the lower half of the lungs indicate that the patient may have an acute problem such as heart failure. The nurse should immediately accomplish further assessments, such as O2 saturation, and notify the HCP. A barrel-shaped chest, and a weak cough effort are associated with aging and immediate action is not indicated. An older patient has less forceful cough and fewer and less functional cilia. Mucous membranes tend to be drier.

The nurse is caring for a client with biliary cancer. The associated jaundice getsprogressively worse. The nurse is most concerned about the potential complication of what symptom?

B. bleeding

A client with jaundice reports fatigue, abdominal pain, loss of appetite, dark urine, nausea, and vomiting. Which pathogen is most likely responsible for the client's condition?

B. hepatitis C virus

A client newly diagnosed with diabetes mellitus is instructed by the primary health care provider to obtain glucagon for emergency home use. The client asks the home care nurse about the purpose of the medication. The nurse should instruct the client that the purpose of the medication is to treat which problem?

B. hypoglycemia from insulin overdose

While undergoing a soapsuds enema, the client reports abdominal cramping. What action should the nurse take?

B. lower the height of the enema bag

A nurse is caring for a client who has a Hemovac portable wound suction device after abdominal surgery. What is the reason why the nurse empties the device when it is half full?

B. negative pressure in the unit lessens as fluid accumulates, interfering with further drainage

The nurse is assessing a client who is experiencing an acute episode of cholecystitis. Which of these clinical manifestations support this diagnosis? Select all that apply

B. pain in the upper right quadrant after a fatty meal E. fever F. complaints of indigestion

Which finding can indicate gallstones and gallbladder obstruction?

B. referred pain to the right shoulder

A client is admitted to the hospital with severe diarrhea, abdominal cramps, and vomiting after eating. These symptoms have lasted 5 days. Upon further assessment, the primary healthcare provider finds that the symptoms occurred after the client ate eggs, salad dressings, and sandwich fillings. Which food borne disease would be suspected in this client?

B. salmonellosis

A client is complaining of low back pain that radiates down the left posterior thigh. The nurse would ask the client if the pain is worsened or aggravated by which factor?

Bending or lifting

A nurse is assessing a client who underwent abdominal surgery 10 days ago. The client complains of pain in the abdomen. What type of pain does the client experience?

B. visceral pain

A home care nurse is visiting a client with a diagnosis of pernicious anemia that developed as a result of gastric surgery. The nurse instructs the client that in this disorder, because the stomach lining produces a decreased amount of a substance known as the intrinsic factor, the client will need which medication?

B. vitamin B12 injections

The nurse teaches a patient about the transmission of pulmonary tuberculosis (TB). Which statement, if made by the patient, indicates that teaching was effective? a. "I will stay indoors whenever possible." b. "My spouse will sleep in the other room." c. "I will take the bus instead of driving." d. "I will keep the windows closed at home."

B."My spouse will sleep in another room" Rationale: Teach the patient how to minimize exposure to close contacts and household members. Homes should be well ventilated, especially the areas where the infected person spends a lot of time. While still infectious, the patient should sleep alone, spend as much time as possible outdoors, and minimize time in congregate settings or on public transportation

The nurse provides discharge teaching for a patient who has two fractured ribs from an automobile accident. Which patient statement indicates that teaching has been effective? A. "I am going to buy a rib binder to wear during the day" B. I should plan on taking the pain pills only at bedtime so I can sleep" C. "I will use the incentive spirometer every hour or two during the day" D. "I can take shallow breaths to prevent my chest from hurting"

C. "I will use the incentive spirometer every hour or two during the day" Rationale: Prevention of the complications of atelectasis and pneumonia is a priority after rib fracture. This can be ensured by deep breathing and coughing. Use of a rib binder, shallow breathing, and taking pain medications only at night are likely to result in atelectasis

The nurse supervises a student nurse who is assigned to take care of a patient with active tuberculosis (TB). Which action, if performed by the student nurse, would require an intervention by the nurse? A. A snack is brought to the patient from the unit refrigerator B. The patient is offered a tissue from the box at the bedside C. A surgical face mask is applied before visiting the patient D. Hand washing is performed before entering the patient's room

C. A surgical face mask is applied before visiting the patient Rationale: A high-efficiency particulate-absorbing (HEPA) mask, rather than a standard surgical mask, should be used when entering the patient's room because the HEPA mask can filter out 100% of small airborne particles. Hand washing before entering the patient's room is appropriate. The student nurse should perform hand washing after handling a tissue that the patient has used, but no precautions are necessary when giving the patient an unused tissue

What risks will the nurse plan to teach a 27-yr-old woman who smokes two packs of cigarettes daily? A. Interstitial cystitis B. Kidney stones C. Bladder cancer D. Bladder infection

C. Bladder cancer Rationale: Cigarette smoking is a risk factor for bladder cancer. The patient's risk for developing interstitial cyclists, urinary tract infection, or kidney stones will not be reduced by quitting smoking

The public health nurse is planning a program to decrease the incidence of meningitis in teenagers and young adults. Which action is most likely to be effective?

Encourage immunization

Which action can the registered nurse (RN) who is caring for a critically ill patient with multiple IV lines and medications delegate to a licensed practical/vocational nurse (LPN/LVN)? A. Verify blood products prior to administration B. Remove the patient's central venous catheter C. Flush a saline lock with normal saline D. Titrate vasoactive IV medications

C. Flush a saline lock with normal saline Rationale: A LPN/LVN has the education, experience, and scope of practice to flush a saline lock with normal saline. Administration of blood products, adjustment of vasoactive infusion rates, and removal of central catheters in critically ill patients require RN level education and scope of practice

Metformin is prescribed for a client with type 2 diabetes mellitus. What is the most common side effect that the nurse should include in the client's teaching plan?

C. Gastrointestinal disturbances

The nurse interviews a patient with a new diagnosis of chronic obstructive pulmonary disease (COPD). Which information is specific in confirming a diagnosis of chronic bronchitis? a. The patient tells the nurse about a family history of bronchitis. b. The patient indicates a 30 pack-year cigarette smoking history. c. The patient reports a productive cough for 3 months every winter. d. The patient denies having respiratory problems until the past 12 months.

C. The patient reports a productive cough for 3 months for every winter Rationale: A diagnosis of chronic bronchitis is based on a history of having a productive cough for 3 months for at least 2 consecutive years. There is no family tendency for chronic bronchitis. Although smoking is the major risk factor for chronic bronchitis, a smoking history does not confirm the diagnosis

Discharge teaching for the patient who underwent laparoscopic cholecystectomy should include the need to:

C. call if there are changes in stool or urine color

The nurse is providing discharge instructions to a client following gastrectomy and should instruct the client to take which measure to assist in preventing dumping syndrome?

C. limit the fluids taken with meals

A nurse is caring for a client with a suspected endocrine tumor that presents with hypertension. Which study will the nurse prepare to monitor that best screens for this condition?

C. metanephrine

The nurse teaches the client about endocrine functioning of Part A. Which information from the client indicates successful learning? (Part A is the pituitary gland.)

C. promotes growth

Treatment modalities for the treatment of cardiogenic shock include:

Cardiac catheterization immediately after insult, vasodilators (nitrate) to reduce afterload, diuretics to reduce preload, beta blockers to reduce heart rate and contractility. Sometimes a circulatory assist device (e.g. intraaortic balloon pump, VAD), and rarely cardiac transplant. NOT TRENDELENBURG - We did not go over the correct answers, but rationale for this would be that the cardiac pump is malfunctioning, and you are increasing the workload if you place the client in trendelenburg.

The nurse is caring for a client being treated for a fat embolus after multiple fractures. Which data would the RN determine is the most favorable indication of resolution of the fat embolus?

Clear mentation

Which client will have the lowest risk for developing hepatotoxicity? A - everolimus B - Azathioprine C - Daclizumab D - Cyclosporine

Client C

Which findings from the analysis of fluid from a patient's right knee arthrocentesis should be of concern to the nurse?

Cloudy fluid

The primary health care provider has determined that a client has contracted hepatitis A based on flu- like symptoms and jaundice. Which statement made by the client supports this medical diagnosis?

D. "I ate shellfish about 2 weeks ago at a local restaurant."

The nurse notes that a patient who was admitted with heart failure has jugular venous distention (JVD) when lying flat in bed. Which follow-up action should the nurse take next? a. Obtain vital signs, including oxygen saturation. B. Have the patient perform the Valsalva maneuver. C. Document this JVD finding in the patient's record. D. Observe for JVD with the patient elevated 45 degrees.

D. Observe for JVD with the patient elevated 45 degrees. Rationale: When the patient is lying flat, the jugular veins are at the level of the right atrium, so JVD is a common (but not a clinically significant) finding. JVD is an expected finding when a patient performs the Valsalva maneuver because right atrial pressure increases. More fluids will further increase any fluid overload. JVD but is not confirmed based on the data given. JVD that persists when the patient is sitting at a 30- to 45-degree angle or greater is significant.

The nurse administers prescribed therapies for a patient with for pulmonate and right-sided heart failure. Which assessment could be used to evaluate the effectiveness of the therapies? A. Palpate for heaves or thrills over the heart B. Auscultate for crackles in the lungs C. Monitor for elevated WBC count D. Observe for distended neck veins

D. Observe for distended neck veins Rationale: Cor pulmonate is right ventricular failure caused by pulmonary hypertension, so clinical manifestations of right ventricular failure such as peripheral edema, jugular vein distention, and right upper-quadrant abdominal tenderness would be expected. Crackles in the lungs are likely to be heard with left-sided heart failure. Findings in cor pulmonate include evidence of right ventricular hypertrophy on electrocardiography and an increase in intensity of the second heart sound. Heaves or thrills are not common with cor pulmonale. WBC count elevation might indicate infection but is not expected with cor pulmonale

The nurse is reviewing the laboratory results for a client with cirrhosis and notes that the ammonia level is 85 mcg/dL (51 mcmol/L). Which dietary selection does the nurse suggest to the client?

D. pasta with sauce

A client is diagnosed with viral hepatitis, complaining of "no appetite" and "losing my taste for food." What instruction should the nurse give the client to provide adequate nutrition?

D. increase intake of fluids, including juices

A client with hiatal hernia chronically experiences heartburn following meals. The nurse should plan to teach the client to avoid which action because it is contraindicated with a hiatal hernia?

D. lying recumbent following meals

A person needs postexposure prophylaxis for hepatitis B with hepatitis B immune globulin (HBIG). When should HBIG be given after exposure?

D. within 24 hours

Allopurinol is prescribed for a patient and the nurse provides medication instruction. Which instruction would the nurse provide to the client?

Drink 3000mL of fluid a day

Which result for a patient with systemic lupus erythematosus (SLE) should the nurse identify as most important to communicate to the HCP?

Elevated blood urea nitrogen (BUN)

After surgery for cancer of the pancreas, the client's nutrition and fluid regimen are influenced by the remaining amount of functioning pancreatic tissue. The nurse considers both the exocrine and the endocrine functions of the pancreas and expects that, postoperatively, the client's dietary regimen will be focused on the management of what substances?

Fats and carbohydrates

Cyclobenzaprine is prescribed. Which disorder would indicate a need to notify HCP?

Glaucoma

A high school teacher with ulnar drift caused by RA is scheduled for arthroplasty to several joints in the hand. Which statement indicates realistic expectation for the surgery?

I will be able to use my fingers with more flexibility to grasp things

A nurse provides care to a client who has a fractured femur after falling from a ladder. Which of the following actions may reduce the incidence of fat emboli?

Immobilize the extremity

A patient has scleroderma manifested by CREST (calcinosis, Raynaud's phenomenon, esophageal dysfunction, sclerodactyly, telangiectasia) syndrome. Which action is included in plan of care?

Keep the environment warm and draft free.

Colchicine is prescribed for a client with a diagnosis of gout. The nurse review the client's record, knowing that this medication would be used with caution in which disorder?

Kidney disease

A patient is being discharged 4 day after hip arthroplasty, using the posterior approach. Which patient action requires intervention by the nurse?

Leaning over the pull on shoes and socks

The nurse is analyzing the lab studies on a client receiving dantrolene to treat muscle spasms from an injury. Which lab test identifies an adverse effect associated with this medication?

Liver function tests

A client admitted to the hospital with chest pain and a history of type 2 diabetes mellitus is scheduled for cardiac catheterization. Which medication would need to be withheld for 24 hours before the procedure and for 48 hours after the procedure

Metformin (Glucophage) needs to be withheld 24 hours before and for 48 hours after cardiac catheterizationwith metformin in the system, the client would be at increased risk for lactic acidosis.

A nurse is receiving end-of-shift report. Which client should the nurse assess first?

New onset confusion in a client who has a left femur fracture

The nurse is reviewing the EKG strip rhythm strip. The p waves and qrs complexes are regular. The pr interval is 0.16 seconds and QRS complexes measure 0.06 second. The overall heart rate is 64 beats/minute. Which would be a correct interpretation based on these characteristics?

Normal sinus rhythm; monitor for any rhythm changes Normal sinus rhythm is defined as a regular rhythm, with an overall rate of 60 to 100 beats/minute. The PR and QRS measurements are normal, measuring 0.12 to 0.20 second and 0.04 to 0.10 second, respectively.

The nurse is conducting teaching to a client in an arm cast about the S/S of compartment syndrome. The nurse determines the client understands the information if they report which early symptom of compartment syndrome?

Numbness and tingling in the fingers

Which menu choice by a patient with osteoporosis indicates the nurse's teaching about appropriate diet is effective

Oatmeal with skim milk and fruit yogurt

Which finding in a patient with a fracture should the nurse immediately report to the HCP

Patient has been incontinent of urine and stool

A client receiving Digoxin daily has a potassium of 3 and reports anorexia. The cardiologist prescribes a serum digoxin level to be done. Which level would the nurse recognize as being outside of the therapeutic range? 2.2 ng/mL

Rationale: The optimal therapeutic range for digoxin is 0.8 to 2.0 ng/mL. If the client is experiencing symptoms such as anorexia and is experiencing hypokalemia as evidenced by a low potassium level, digoxin toxicity is a concern.

The nurse is caring for a client who had an above the knee amputation 2 days ago. The residual limb was wrapped with an elastic compression bandage which has come off. What do you do?

Rewrap the residual limb with an elastic compression bandage

A client with diabetes mellitus has had a right below the knee amputation. Given the client's history of diabetes mellitus, which complication is the client at most risk for after the surgery?

Separation of the wound edges

The client has developed a fib with a ventricular rate of 150 beats per minute. Which associated findings would the nurse anticipate in the assessment?

Syncope Dizziness Palpitations The client with uncontrolled atrial fibrillation with a ventricular rate greater than 100 beats/min is at risk for low cardiac output due to loss of atrial kick. The nurse assesses the client for palpitations, chest pain or discomfort, hypotension, pulse deficit, fatigue, weakness, dizziness, syncope, shortness of breath, and distended neck veins.

Client is on treatment for RA for 3 weeks. During administration of etanercept, what is most important for the nurse to assess?

The white blood cell counts and platelet counts

Which information obtained during the nurse's assessment may indicate a patient's increased risk for musculoskeletal problems?

The patient is 5ft 2in tall and weighs 180 lbs

A patient arrived to the ED after tripping over a rug and falling at home. Which finding is most important to report to the HCP?

The right arm appears to be shorter than the left

The nurse is caring for a client with a diagnosis of gout. Which lab value would the nurse expect to note in the client?

Uric acid level of 9.0 mg/dL

Which discharge instructions should the emergency department nurse include for a patient with a sprained ankle?

Use pillows to elevate the ankle above the heart.

When using evidence-based practice, the nurse:

Uses clinical decision making and judgement

Which patient is most appropriate for the intensive care unit (ICU) charge nurse to assign to a registered nurse (RN) who has floated from the medical unit? a. A 45-year-old receiving IV antibiotics for meningococcal meningitis b. A 25-year-old admitted with a skull fracture and craniotomy the previous day c. A 55-year-old who has increased intracranial pressure (ICP) and is receiving hyperventilation therapy d. A 35-year-old with ICP monitoring after a head injury last week

a. A 45-year-old receiving IV antibiotics for meningococcal meningitis

Which patient should the nurse assign as the roommate for a patient who has aplastic anemia? a. A patient with chronic heart failure b. A patient who has viral pneumoniac. A patient who has right leg cellulitisd. A patient with multiple abdominal drains

a. A patient with chronic heart failure

A client has just had a hemorrhoidectomy. Which nursing interventions are appropriate for this client? Select all that apply

a. Administer stool softeners as prescribed b. Encourage a high-fiber diet to promote bowel movements without straining c. Apply cold packs to the anal-rectal area over the dressing until the packing is removed

The nurse is caring for a young adult client who had a mandatory electrocardiogram (ECG) before participating on a college swim team and is found to have sinus bradycardia, rate 52. BP is 114/54 mm Hg, and the student denies any health problems. Which of the following actions by the nurse is best? a. Allow the student to participate on the swim team. b. Refer the student to a cardiologist for further assessment.c. Obtain more detailed information about the student's health history. d. Tell the student to stop swimming immediately if any dyspnea occurs.

a. Allow the student to participate on the swim team. Rationale: In an aerobically trained individual, sinus bradycardia is normal.

A 40-year-old patient has a ruptured cerebral aneurysm and subarachnoid hemorrhage. Which intervention will be included in the care plan? a. Apply intermittent pneumatic compression stockings. b. Assist to dangle on edge of bed and assess for dizziness. c. Encourage patient to cough and deep breathe every 4 hours. d. Insert an oropharyngeal airway to prevent airway obstruction.

a. Apply intermittent pneumatic compression stockings. The patient with a subarachnoid hemorrhage usually has minimal activity to prevent cerebral vasospasm or further bleeding and is at risk for venous thromboembolism (VTE).

A patient who has a history of chronic obstructive pulmonary disease (COPD) was hospitalized for increasing shortness of breath and chronic hypoxemia (SaO2 levels of 89% to 90%). In planning for discharge, which action by the nurse will be most effective in improving compliance with discharge teaching? a. Arrange for the patient's caregiver to be present during the teaching. b. Accomplish the patient teaching just before the scheduled discharge. c. Start giving the patient discharge teaching during the admission process. d. Have the patient repeat the instructions immediately after teaching.

a. Arrange for the patient's caregiver to be present during the teaching. Rationale: Hypoxemia interferes with the patient's ability to learn and retain information, so having the patient's caregiver present will increase the likelihood that discharge instructions will be followed. Having the patient repeat the instructions will indicate that the information is understood at the time, but it does not guarantee retention of the information. Because the patient is likely to be distracted just before discharge, giving discharge instructions just before discharge is not ideal. The patient is likely to be anxious and even more hypoxemic than usual on the day of admission, so teaching about discharge should be postponed.

As the nurse admits a patient in end-stage kidney disease to the hospital, the patient tells the nurse, If my heart or breathing stop, I do not want to be resuscitated. Which action is best for the nurse to take? a. Ask if these wishes have been discussed with the health care provider. b. Place a Do Not Resuscitate (DNR) notation in the patients care plan. c. Inform the patient that a notarized advance directive must be included in the record or resuscitation must be performed. d. Advise the patient to designate a person to make health care decisions when the patient is not able to make them independently.

a. Ask if these wishes have been discussed with the health care provider.

While assessing the health of a female client, the nurse suspects endocrine dysfunction. Which findings support the nurse's suspicion? Select all that apply

a. Blood pressure of 172/80 mm Hg b. Protrusion of eyeballs c. Diminished pubic hair

A nurse assesses a postoperative patient 2 days after chest surgery. What findings indicate that the patient requires better pain management (select all that apply)? a. Confusion b. Hypoglycemia c. Poor cough effort d. Shallow breathing e. Elevated temperature

a. Confusion c. Poor cough effort d. Shallow breathing e. Elevated temperature

A 12-year-old child with type 2 diabetes is scheduled for abdominal surgery. Which factors are most important for the nurse to consider during the postoperative period? Select all that apply.

a. Diabetic control is usually maintained with insulin after surgery b. The blood glucose level will increase because of stress of surgery

The nurse plans a presentation for community members about how to decrease the risk for antibiotic- resistant infections. Which information will the nurse include in the teaching plan (select all that apply)? a. Continue taking antibiotics until all the medication is gone. b. Antibiotics may sometimes be prescribed to prevent infection. c. Unused antibiotics that are more than a year old should be discarded.d. Antibiotics are effective in treating influenza associated with high fevers. e. Hand washing is effective in preventing many viral and bacterial infections.

a. Continue taking antibiotics until all the medication is gone. b. Antibiotics may sometimes be prescribed to prevent infection. e. Hand washing is effective in preventing many viral and bacterial infections.

A patient develops neutropenia after receiving chemotherapy. Which information about ways to prevent infection will the nurse include in the teaching plan (select all that apply)? a. Cook food thoroughly before eating. b. Choose low fiber, low residue foods. c. Avoid public transportation such as buses. d. Use rectal suppositories if needed for constipation. e. Talk to the oncologist before having any dental work done.

a. Cook food thoroughly before eating. c. Avoid public transportation such as buses. e. Talk to the oncologist before having any dental work done.

The nurse admits a terminally ill patient to the hospital. What is the first action that the nurse should complete when planning this patients care? a. Determine the patients wishes regarding end-of-life care. b. Emphasize the importance of addressing any family issues. c. Discuss the normal grief process with the patient and family. d. Encourage the patient to talk about any fears or unresolved issues.

a. Determine the patients wishes regarding end-of-life care.

A patient who has bacterial meningitis is disoriented and anxious. Which nursing action will be included in the plan of care? a. Encourage family members to remain at the bedside. b. Apply soft restraints to protect the patient from injury. c. Keep the room well-lighted to improve patient orientation. d. Minimize contact with the patient to decrease sensory input.

a. Encourage family members to remain at the bedside. Patients with meningitis and disorientation will be calmed by the presence of someone familiar at the bedside. Restraints should be avoided because they increase agitation and anxiety. The patient requires frequent assessment for complications; the use of touch and a soothing voice will decrease anxiety for most patients. The patient will have photophobia, so the light should be dim.

A patient with late-stage cirrhosis develops portal hypertension. Which complications can develop from this condition? (select all that apply)

a. Esophageal varices b. ascites

The patient diagnosed with pancreatic cancer underwent a Whipple procedure 2 days ago. Which clinical problem has the highest priority?

a. Fluid volume imbalance

The nurse cares for a terminally ill patient who is experiencing pain that is continuous and severe. How should the nurse schedule the administration of opioid pain medications? a. Give around-the-clock routine administration of analgesics. b. Provide PRN doses of medication whenever the patient requests. c. Offer enough pain medication to keep the patient sedated and unaware of stimuli. d. Suggest analgesic doses that provide pain control without decreasing respiratory rate.

a. Give around-the-clock routine administration of analgesics.

The nurse cares for a patient infected with human immunodeficiency virus (HIV) who has just been diagnosed with asymptomatic chronic HIV infection. Which prophylactic measures will the nurse include in the plan of care (select all that apply)? a. Hepatitis B vaccine b. Pneumococcal vaccine c. Influenza virus vaccine d. Trimethoprim-sulfamethoxazole e. Varicella zoster immune globulin

a. Hepatitis B vaccine b. Pneumococcal vaccine c. Influenza virus vaccine

Four days after abdominal surgery a client has not passed flatus and there are no bowel sounds. Paralytic ileus is suspected. What does the nurse conclude is the most likely cause of the ileus?

a. Impaired neural functioning

A patient has been taking phenytoin (Dilantin) for 2 years. Which action will the nurse take when evaluating for adverse effects of the medication? a. Inspect the oral mucosa. b. Listen to the lung sounds. c. Auscultate the bowel tones. d. Check pupil reaction to light.

a. Inspect the oral mucosa.

Which intervention would the nurse expect for a patient admitted with acute pancreatitis?

a. Keep the patient NPO

Which actions will the nurse include in the plan of care for a patient with metastatic bone cancer of the left femur (select all that apply)? a. Monitor serum calcium. b. Teach about the need for strict bed rest. c. Explain the use of sustained-release opioids. d. Support the left leg when repositioning the patient. e. Support family and patient as they discuss the prognosis.

a. Monitor serum calcium. c. Explain the use of sustained-release opioids. d. Support the left leg when repositioning the patient. e. Support family and patient as they discuss the prognosis. The nurse will monitor for hypercalcemia caused by bone decalcification. Support of the leg helps reduce the risk for pathologic fractures. Although the patient may be reluctant to exercise, activity is important to maintain function and avoid complications associated with immobility. Adequate pain medication, including sustained-release and rapid-acting opioids, is needed for the severe pain often associated with bone cancer. The prognosis for metastatic bone cancer is poor so the patient and family need to be supported as they deal with the reality of the situation.

A patient with an open leg leison has a white blood cell (WBC) count of 13, 500/L and a band count of 11%. What action should the nurse take first? a. Obtain wound cultures. b. Start antibiotic therapy. c. Redress the wound with wet-to-dry dressings. d. Continue to monitor the wound for purulent drainage.

a. Obtain wound cultures.

Which menu choice indicates that the patient understands the nurses teaching about best dietary choices for iron-deficiency anemia? a. Omelet and whole wheat toast b. Cantaloupe and cottage cheese c. Strawberry and banana fruit plate d. Cornmeal muffin and orange juice

a. Omelet and whole wheat toast

The nurse at the clinic is interviewing a 64-year-old woman who is 5 feet, 3 inches tall and weighs 125 pounds (57 kg). The patient has not seen a health care provider for 20 years. She walks 5 miles most days and has a glass of wine 2 or 3 times a week. Which topics will the nurse plan to include in patient teaching about cancer screening and decreasing cancer risk (select all that apply)? a. Pap testing b. Tobacco use c. Sunscreen use d. Mammography e. Colorectal screening

a. Pap testing c. Sunscreen use d. Mammography e. Colorectal screening

A young adult patient with cystic fibrosis (CF) is admitted to the hospital with increased dyspnea. Which intervention should the nurse include in the plan of care? a. Perform chest physiotherapy every 4 hours. b. Place the patient on a low-sodium diet. c. Schedule a sweat chloride test. d. Arrange for a hospice nurse visit.

a. Perform chest physiotherapy every 4 hours. Rationale: Routine scheduling of airway clearance techniques is an essential intervention for patients with CF. A sweat chloride test is used to diagnose CF, but it does not provide any information about the effectiveness of therapy. There is no indication that the patient is terminally ill. Patients with CF lose excessive sodium in their sweat and require high amounts of dietary sodium.

Which nursing actions for the care of a dying patient can the nurse delegate to a licensed practical/vocational nurse (LPN/LVN) (select all that apply)? a. Provide postmortem care to the patient. b. Encourage the family members to talk with and reassure the patient. c. Determine how frequently physical assessments are needed for the patient. d. Teach family members about commonly occurring signs of approaching death e. Administer the prescribed morphine sulfate sublingual as necessary for pain control.

a. Provide postmortem care to the patient. b. Encourage the family members to talk with and reassure the patient. e. Administer the prescribed morphine sulfate sublingual as necessary for pain control.

What finding should indicate to the nurse that colchicine has been effective for a patient with an acute attack of gout? a. Reduced joint pain b. Increased urine output c. Elevated serum uric acid d. Increased white blood cells

a. Reduced joint pain Colchicine reduces joint pain in 24 to 48 hours by decreasing inflammation. The recommended increase in fluid intake of 2 to 3 L/day during acute gout would increase urine output but would not indicate the effectiveness of colchicine. Elevated serum uric acid would result in increased symptoms. The WBC count might decrease with decreased inflammation but would not increase.

A patient with a family history of cystic fibrosis (CF) asks for information about genetic testing. Which response by the nurse is most appropriate? a. Refer the patient to a qualified genetic counselor. b. Ask the patient why genetic testing is so important. c. Remind the patient that genetic testing has many social implications. d. Tell the patient that cystic fibrosis is an autosomal recessive disorder.

a. Refer the patient to a qualified genetic counselor.

A nurse in the postanesthesia care unit (PACU) is providing care to a client who had an abdominal cholecystectomy and observes serosanguineous drainage on the abdominal dressing. What is the next nursing action?

a. Reinforce the dressing

The primary healthcare provider prescribed a diagnostic study with contrast medium for an older adult who has an endocrine disorder. Which assessment result should the nurse check before the study?

a. Serum creatinine

A 25-yr-old patient has been admitted with a severe crushing injury after an industrial accident. Which laboratory result will be most important to report to the health care provider? a. Serum potassium level of 6.5 mEq/L b. Blood urea nitrogen (BUN) of 56 mg/dL c. Serum creatinine level of 2.1 mg/dL d. White blood cell count of 11,500/µL

a. Serum potassium level of 6.5 mEq/L Rationale: The hyperkalemia associated with crushing injuries may cause cardiac arrest and should be treated immediately. The nurse will report the other laboratory values, but abnormalities in these are not immediately life threatening.

A hospitalized 31-year-old patient with a history of cluster headache awakens during the night with a severe stabbing headache. Which action should the nurse take first? a. Start the ordered PRN oxygen at 6 L/min. b. Put a moist hot pack on the patient's neck. c. Give the ordered PRN acetaminophen (Tylenol). d. Notify the patient's health care provider immediately.

a. Start the ordered PRN oxygen at 6 L/min.

A patient in the outpatient clinic has a new diagnosis of peripheral artery disease (PAD). Which group of drugs will the nurse plan to include when teaching about PAD management? a. Statins b. Antibiotics c. Thrombolytics d. Anticoagulants

a. Statins Rationale: Research indicates that statin use by patients with PAD improves multiple outcomes. There is no research that supports the use of the other drug categories in PAD.

The nurse anticipates the preferred treatment for a patient with acute hepatitis A infection will include

a. Supportive care

Which action will the nurse take when caring for a patient with osteomalacia? a. Teach about the use of vitamin D supplements. b. Educate about the need for weight-bearing exercise. c. Discuss the use of medications such as bisphosphonates. d. Emphasize the importance of sunscreen use when outside.

a. Teach about the use of vitamin D supplements. Osteomalacia is caused by inadequate intake or absorption of vitamin D. Weight-bearing exercise and bisphosphonate administration may be used for osteoporosis but will not be beneficial for osteomalacia. Because ultraviolet light is needed for the body to synthesize vitamin D, the patient might be taught that 20 minutes a day of sun exposure is beneficial.

A client has a tentative diagnosis of primary biliary cirrhosis. What skin change does the nurse expect to observe when performing a physical assessment?

a. Telangiectasia

A female patient with a suspected urinary tract infection is to provide a clean-catch urine specimen for culture and sensitivity testing. What should the nurse do to obtain the specimen? a. Tell the patient to clean the urethral area, void a small amount into the toilet, then void directly into a sterile container. b. Have the patient empty the bladder completely; then obtain the next urine specimen that the patient is able to void. c. Insert a short sterile "mini" catheter attached to a collecting container into the urethra and bladder to obtain the specimen. d. Clean the area around the patient's meatus with a povidone iodine (Betadine) swab and then have the patient void into a sterile specimen cup.

a. Tell the patient to clean the urethral area, void a small amount into the toilet, then void directly into a sterile container. Rationale: This answer describes the technique for obtaining a clean-catch specimen. The answer beginning, "insert a short, small, 'mini' catheter attached to a collecting container" describes a technique that would result in a sterile specimen, but a health care provider's order for a catheterized specimen would be required. Using Betadine before obtaining the specimen might result in suppressing the growth of some bacteria. The technique described in the answer beginning "have the patient empty the bladder completely" would not result in a sterile specimen.

A patient who is 2 days post femoral popliteal bypass graft to the right leg is being cared for on the vascular unit. Which action by a licensed practical/vocational nurse (LPN/LVN) caring for the patient requires the registered nurse (RN) to intervene? a. The LPN/LVN has the patient to sit in a chair for 2 hours. b. The LPN/LVN gives the prescribed aspirin after breakfast. c. The LPN/LVN assists the patient to walk 40 feet in the hallway. d. The LPN/LVN places the patient in Fowler's position for meals.

a. The LPN/LVN has the patient to sit in a chair for 2 hours.

A patient is being evaluated for a possible spinal cord tumor. Which findings should the nurse recognize as needing the most immediate action? a. The patient has new onset weakness of both legs. b. The patient complains of chronic severe back pain. c. The patient starts to cry and says, "I feel hopeless." d. The patient expresses anxiety about having surgery.

a. The patient has new onset weakness of both legs. The new onset of symptoms indicates cord compression, which is an emergency that requires rapid treatment to avoid permanent loss of function. The other patient assessments also indicate a need for nursing action but do not require intervention as rapidly as the new onset weakness.

When admitting a 42-year-old patient with a possible brain injury after a car accident to the emergency department (ED), the nurse obtains the following information. Which finding is most important to report to the health care provider? a. The patient takes warfarin (Coumadin) daily. b. The patient's blood pressure is 162/94 mm Hg. c. The patient is unable to remember the accident. d. The patient complains of a severe dull headac

a. The patient takes warfarin (Coumadin) daily.

When caring for a patient who experienced a T2 spinal cord transection 24 hours ago, which collaborative and nursing actions will the nurse include in the plan of care (select all that apply)? a. Urinary catheter care b. Nasogastric (NG) tube feeding c. Continuous cardiac monitoring d. Maintain a warm room temperature e. Administration of H2 receptor blockers

a. Urinary catheter care c. Continuous cardiac monitoring d. Maintain a warm room temperature e. Administration of H2 receptor blockers The patient is at risk for bradycardia and poikilothermia caused by sympathetic nervous system dysfunction and should have continuous cardiac monitoring and maintenance of a relatively warm room temperature. Gastrointestinal (GI) motility is decreased initially and NG suctioning is indicated. To avoid bladder distention, a urinary retention catheter is used during this acute phase. Stress ulcers are a common complication, but can be avoided through the use of the H2 receptor blockers such as famotidine

A client with biliary cirrhosis receives serum albumin therapy. What is the most effective method for the nurse to evaluate the client's response to therapy?

a. Weigh daily

An older adult is receiving standard multidrug therapy for tuberculosis (TB). Which finding should the nurse report to the health care provider? a. Yellow-tinged sclera b. Difficulty hearing high-pitched voices c. Orange-colored sputum d. Thickening of the fingernails

a. Yellow-tinged sclera Rationale: Noninfectious hepatitis is a toxic effect of isoniazid, rifampin, and pyrazinamide, and patients who develop hepatotoxicity will need to use other medications. Changes in hearing and nail thickening are not expected with the four medications used for initial TB drug therapy. Presbycusis is an expected finding in the older adult patient. Orange discoloration of body fluids is an expected side effect of rifampin and not an indication to call the health care provider.

A patient is admitted to the hospital with possible acute pericarditis. The nurse should plan to teach the patient about the purpose of: a. echocardiography. b. daily blood cultures. c. cardiac catheterization. d. 24-hour Holter monitor.

a. echocardiography. Echocardiograms are useful in detecting the presence of the pericardial effusions associated with pericarditis. Blood cultures are not indicated unless the patient has evidence of sepsis. Cardiac catheterization and 24-hour Holter monitor are not diagnostic procedures for pericarditis.

A registered nurse is teaching a nursing student about the proper techniques of an abdominal assessment. Which order of assessment stated by the nursing student indicates effective learning?

a."I will inspect the surface motion of the abdomen" b. "I will note the position of the umbilicus" c. "I will assess for bowel motility by auscultation" d. "I will palpate to assess for any abdominal tenderness"

To assess for functional deficits, which question will the nurse ask a patient who has been admitted for treatment of a benign occipital lobe tumor? a. "Do you have difficulty in hearing?" b. "Are you experiencing visual problems?" c. "Are you having any trouble with your balance?" d. "Have you developed any weakness on one side?"

b. "Are you experiencing visual problems?"

The registered nurse instructs the new nurse in orientation regarding the physiologic processes of the endocrine system prior to client assessment. Which statement made by the new nurse indicates effective learning?

b. "The hormones of the endocrine system exert their action by 'lock and key' mechanism."

A patient with cancer that has metastasized to the liver has symptoms of fluid retention, including edema and ascites. To determine the effectiveness of diuretic therapy on ascites, what should the nurse assess?

b. Abdominal girth

The nurse is caring for a patient who is recovering from a sudden cardiac death (SCD) event and has no evidence of an acute myocardial infarction (AMI). What should the nurse anticipate teaching the patient? a. Sudden cardiac death events rarely reoccur. b. Additional diagnostic testing will be required. c. Long-term anticoagulation therapy will be needed. d. Limiting physical activity will prevent future SCD events.

b. Additional diagnostic testing will be required. Rationale: Diagnostic testing (e.g., stress test, Holter monitor, electrophysiologic studies, cardiac catheterization) is used to determine the possible cause of the SCD and treatment options. SCD is likely to recur. Anticoagulation therapy will not have any effect on the incidence of SCD, and SCD can occur even when the patient is resting.

A patient with terminal cancer-related pain and a history of opioid abuse complains of breakthrough pain 2 hours before the next dose of sustained-release morphine sulfate (MS Contin) is due. Which action should the nurse take first? a. Use distraction by talking about things the patient enjoys. b. Administer the prescribed PRN immediate-acting morphine. c. Suggest the use of alternative therapies such as heat or cold. d. Consult with the doctor about increasing the MS Contin dose.

b. Administer the prescribed PRN immediate-acting morphine.

The nurse is evaluating a client's response to cardioversion. Which observation would be of highest priority to the nurse? a. Blood pressure b. Airway patency c. Oxygen flow rate d. Level of consciousness

b. Airway patency Nursing responsibilities after cardioversion include maintenance first of a patent airway, and then oxygen administration, assessment of vital signs and level of consciousness, and dysrhythmia detection.

A patient tells the nurse, I would like to use a home genetic test to see if I will develop breast cancer. Which response by the nurse is best? a. Home genetic testing is very expensive. b. Are you concerned about developing breast cancer? c. Wont you be depressed if the testing shows a positive result? d. Genetic testing can only determine if you are at higher risk for breast cancer.

b. Are you concerned about developing breast cancer?

A patient who is anxious and has difficulty breathing seeks treatment after being stung by a wasp. What is the nurses priority action? a. Have the patient lie down. b. Assess the patient's airway. c. Administer high-flow oxygen. d. Remove the stinger from the site.

b. Assess the patient's airway.

Which nursing action has the highest priority for a patient who was admitted 16 hours previously with a C5 spinal cord injury? a. Cardiac monitoring for bradycardia b. Assessment of respiratory rate and effort c. Application of pneumatic compression devices to legs d. Administration of methylprednisolone (Solu-Medrol) infusion

b. Assessment of respiratory rate and effort Edema around the area of injury may lead to damage above the C4 level, so the highest priority is assessment of the patient's respiratory function. Methylprednisolone (Solu-Medrol) is no longer recommended for the treatment of spinal cord injuries. The other actions also are appropriate but are not as important as assessment of respiratory effort.

Several weeks after a stroke, a 50-year-old male patient has impaired awareness of bladder fullness, resulting in urinary incontinence. Which nursing intervention will be best to include in the initial plan for an effective bladder training program? a. Limit fluid intake to 1200 mL daily to reduce urine volume. b. Assist the patient onto the bedside commode every 2 hours. c. Perform intermittent catheterization after each voiding to check for residual urine. d. Use an external "condom" catheter to protect the skin and prevent embarrassment.

b. Assist the patient onto the bedside commode every 2 hours. Developing a regular voiding schedule will prevent incontinence and may increase patient awareness of a full bladder. A 1200 mL fluid restriction may lead to dehydration. Intermittent catheterization and use of a condom catheter are appropriate in the acute phase of stroke, but should not be considered solutions for long-term management because of the risks for urinary tract infection (UTI) and skin breakdown.

Nursing staff on a hospital unit are reviewing rates of health care-associated infections (HAI) of the urinary tract. Which nursing action will be most helpful in decreasing the risk for urinary HAI in patients admitted to the hospital? a. Encouraging adequate oral fluid and nutritional intake b. Avoiding unnecessary urinary catheterization c. Providing perineal hygiene to patients daily and as needed d. Testing urine with a dipstick daily for nitrites

b. Avoiding unnecessary urinary catheterization Rationale: Because catheterization bypasses many of the protective mechanisms that prevent urinary tract infection (UTI), avoidance of catheterization is the most effective means of reducing HAI. The other actions will also be helpful but are not as useful as decreasing urinary catheter use.

A patient with pancytopenia of unknown origin is scheduled for the following diagnostic tests. The nurse will provide a consent form to sign for which test? a. ABO blood typing b. Bone marrow biopsy c. Abdominal ultrasoundd. Complete blood count (CBC)

b. Bone marrow biopsy

A 20-year-old male patient is admitted with a head injury after a collision while playing football. After noting that the patient has developed clear nasal drainage, which action should the nurse take? a. Have the patient gently blow the nose. b. Check the drainage for glucose content. c. Teach the patient that rhinorrhea is expected after a head injury. d. Obtain a specimen of the fluid to send for culture and sensitivity

b. Check the drainage for glucose content. Clear nasal drainage in a patient with a head injury suggests a dural tear and cerebrospinal fluid (CSF) leakage. If the drainage is CSF, it will test positive for glucose. Fluid leaking from the nose will have normal nasal flora, so culture and sensitivity will not be useful. Blowing the nose is avoided to prevent CSF leakage.

Which nursing action could the nurse delegate to unlicensed assistive personnel (UAP) when caring for a patient who is using a fentanyl (Duragesic) patch and a heating pad for treatment of chronic back pain? a. Check the skin under the heating pad. b. Count the respiratory rate every 2 hours. c. Monitor sedation using the sedation assessment scale. d. Ask the patient about whether pain control is effective.

b. Count the respiratory rate every 2 hours.

The nurse notes crackling sounds and a grating sensation with palpation of an older patient's elbow. How will this finding be documented? a. Torticollis b. Crepitation c. Subluxation d. Epicondylitis

b. Crepitation

A hospice nurse who has become close to a terminally ill patient is present in the home when the patient dies and feels saddened and tearful as the family members begin to cry. Which action should the nurse take at this time? a. Contact a grief counselor as soon as possible. b. Cry along with the patients family members. c. Leave the home as soon as possible to allow the family to grieve privately. d. Consider whether working in hospice is desirable because patient losses are common.

b. Cry along with the patients family members.

A client who had abdominal surgery is receiving patient-controlled analgesia intravenously to manage pain. The pump is programmed to deliver a basal dose and bolus doses that can be accessed by the client, with a lock-out time frame of 10 minutes. The nurse assesses use of the pump during the last hour and identifies that the client attempted to self-administer the analgesic 10 times. Further assessment reveals that the client is experiencing pain still. What should the nurse do first?

b. Determine the integrity of the intravenous delivery system

Which action will the public health nurse take to reduce the incidence of epidemic encephalitis in a community? a. Teach about prophylactic antibiotics after exposure to encephalitis. b. Encourage the use of effective insect repellent during mosquito season. c. Remind patients that most cases of viral encephalitis can be cared for at home. d. Arrange to screen school-age children for West Nile virus during the school year.

b. Encourage the use of effective insect repellent during mosquito season. Mosquitoes and ticks usually spread epidemic encephalitis. Use of insect repellent is effective in reducing risk. Encephalitis frequently requires that the patient be hospitalized in an intensive care unit during the initial stages. Antibiotic prophylaxis is not used to prevent encephalitis because most encephalitis is viral. West Nile virus is most common in adults over age 50 during the summer and early fall.

According to the Center for Disease Control (CDC) guidelines, which personal protective equipment will the nurse put on when assessing a patient who is on contact precautions for diarrhea caused by Clostridium difficile(select all that apply)? a. Mask b. Gown c. Gloves d. Shoe covers e. Eye protection

b. Gown c. Gloves

Which action will the nurse in the hypertension clinic take to obtain an accurate baseline blood pressure (BP) for a new patient? a. Deflate the BP cuff at a rate of 5 to 10 mm Hg per second. b. Have the patient sit in a chair with the feet flat on the floor. c. Assist the patient to the supine position for BP measurements. d. Obtain two BP readings in the dominant arm and average the results.

b. Have the patient sit in a chair with the feet flat on the floor

Which nursing action will be most effective in ensuring daily medication compliance for a patient with mild dementia? a. Setting the medications up monthly in a medication box b. Having the patient's family member administer the medication c. Posting reminders to take the medications in the patient's house d. Calling the patient weekly with a reminder to take the medication

b. Having the patient's family member administer the medication

The nurse is reviewing laboratory results and notes an aPTT level of 28 seconds. The nurse should notify the health care provider in anticipation of adjusting which medication? a. Aspirin b. Heparin c. Warfarin d. Erythropoietin

b. Heparin

Which question asked by the nurse will give the most information about the patients metastatic bone cancer pain? a. How long have you had this pain? b. How would you describe your pain? c. How much medication do you take for the pain?d. How many times a day do you take medication for the pain?

b. How would you describe your pain?

Which nursing action should be included in the plan of care after endovascular repair of an abdominal aortic aneurysm? a. Record hourly chest tube drainage. b. Monitor fluid intake and urine output. c. Check the abdominal incision for any redness. d. Teach the reason for a prolonged recovery period.

b. Monitor fluid intake and urine output. Because renal artery occlusion can occur after endovascular repair, the nurse should monitor parameters of renal function such as intake and output. Chest tubes will not be needed for endovascular surgery, the recovery period will be short, and there will not be an abdominal wound.

The health care provider orders a patient-controlled analgesia (PCA) machine to provide pain relief for a patient with acute surgical pain who has never received opioids in the past. Which nursing actions regarding opioid administration are appropriate at this time (select all that apply)? a. Assess for signs that the patient is becoming addicted to the opioid. b. Monitor for therapeutic and adverse effects of opioid administration. c. Emphasize that the risk of some opioid side effects increases over time. d. Teach the patient about how analgesics improve postoperative activity levels. e. Provide instructions on decreasing opioid doses by the second postoperative day.

b. Monitor for therapeutic and adverse effects of opioid administration. d. Teach the patient about how analgesics improve postoperative activity levels.

The nurse obtains the following info newly diagnosed with elevated blood pressure. Which finding is most important to address with the patient? a. Low dietary fiber intake b. No regular physical exercise c. Drinks a beer with dinner every night d. Weight is 5 pounds above ideal weight

b. No regular physical exercise Rationale: The recommendations for preventing hypertension include exercising aerobically for 30 minutes most days of the week. A weight that is 5 pounds over the ideal body weight is not a risk factor for hypertension. The Dietary Approaches to Stop Hypertension (DASH) diet is high in fiber, but increasing fiber alone will not prevent hypertension from developing. The patient's alcohol intake is within guidelines and will not increase the hypertension risk.

The nurse is admitting a patient newly diagnosed with peripheral artery disease. Which admission order should the nurse question? a. Cilostazol drug therapy b. Omeprazole drug therapy c. Use of treadmill for exercised. Exercise to the point of discomfort

b. Omeprazole drug therapy Rationale: Because the antiplatelet effect of clopidogrel is reduced when it is used with omeprazole, the nurse should clarify this order with the health care provider. The other interventions are appropriate for a patient with peripheral artery disease.

The nurse who works in the vascular clinic has several patients with venous insufficiency scheduled today. Which patient should the nurse assign to an experienced licensed practical/vocational nurse (LPN/LVN)? a. Patient who has been complaining of increased edema and skin changes in the legs b. Patient who needs wound care for a chronic venous stasis ulcer on the right lower leg c. Patient who has a history of venous thromboembolism and is complaining of dyspnea d. Patient who needs teaching about elastic compression stockings for venous insufficiency

b. Patient who needs wound care for a chronic venous stasis ulcer on the right lower leg

It is important for the nurse providing care for a patient with sickle cell crisis to a. limit the patients intake of oral and IV fluids. b. evaluate the effectiveness of opioid analgesics. c. encourage the patient to ambulate as much as tolerated. d. teach the patient about high-protein, high-calorie foods.

b. evaluate the effectiveness of opioid analgesics.

Which intervention will the nurse include in the plan of care for a patient with moderate dementia who had an appendectomy 2 days ago? a. Provide complete personal hygiene care for the patient. b. Remind the patient frequently about being in the hospital. c. Reposition the patient frequently to avoid skin breakdown. d. Place suction at the bedside to decrease the risk for aspiration.

b. Remind the patient frequently about being in the hospital.

A client with a family history of diabetes is concerned about the effects of psychiatric medication on the endocrine system. Which psychotropic medication is most likely to cause metabolic syndrome?

b. Risperidone

The nurse provides discharge instructions to a patient who has an immune deficiency involving the T lymphocytes. Which screening should the nurse include in the teaching plan for this patient? a. Screening for allergies b. Screening for cancers c. Antibody deficiency screening d. Screening for autoimmune disorders

b. Screening for cancers

The nurse prepares a patient who has a left-sided pleural effusion for a thoracentesis. How should the nurse position the patient? a. On the right side with the left arm extended above the head b. Sitting upright with the arms supported on an over bed table c. Supine with the head of the bed elevated 30 degrees d. High-Fowler's position with the left arm extended

b. Sitting upright with the arms supported on an over bed table Rationale: The upright position with the arms supported increases lung expansion, allows fluid to collect at the lung bases, and expands the intercostal space so that access to the pleural space is easier. The other positions would increase the work of breathing for the patient and make it more difficult for the health care provider performing the thoracentesis.

A patient with renal failure is on a low phosphate diet. Which food should the nurse instruct unlicensed assistive personnel (UAP) to remove from the patient's food tray? a. Grape juice b. Skim milk c. Mixed green salad d. Fried chicken breast

b. Skim milk Rationale: Foods high in phosphate include milk and other dairy products, so these are restricted on low-phosphate diets. Green, leafy vegetables; high-fat foods; and fruits and juices are not high in phosphate and are not restricted.

The registered nurse (RN) caring for an HIV-positive patient admitted with tuberculosis can delegate which action to unlicensed assistive personnel (UAP)? a. Teach the patient about how to use tissues to dispose of respiratory secretions. b. Stock the patients room with all the necessary personal protective equipment. c. Interview the patient to obtain the names of family members and close contacts. d. Tell the patients family members the reason for the use of airborne precautions.

b. Stock the patients room with all the necessary personal protective equipment.

A 73-year-old patient with Parkinson's disease has a nursing diagnosis of impaired physical mobility related to bradykinesia. Which action will the nurse include in the plan of care? a. Instruct the patient in activities that can be done while lying or sitting. b. Suggest that the patient rock from side to side to initiate leg movement. c. Have the patient take small steps in a straight line directly in front of the feet. d. Teach the patient to keep the feet in contact with the floor and slide them forward.

b. Suggest that the patient rock from side to side to initiate leg movement. Rocking the body from side to side stimulates balance and improves mobility. The patient will be encouraged to continue exercising because this will maintain functional abilities. Maintaining a wide base of support will help with balance. The patient should lift the feet and avoid a shuffling gait

After evacuation of an epidural hematoma, a patient's intracranial pressure (ICP) is being monitored with an intraventricular catheter. Which information obtained by the nurse requires urgent communication with the health care provider? a. Pulse of 102 beats/min b. Temperature of 101.6°F c. Intracranial pressure of 15 mm Hg d. Mean arterial pressure of 90 mmHg

b. Temperature of 101.6°F

The health care provider is considering the use of sumatriptan (Imitrex) for a 54-year-old male patient with migraine headaches. Which information obtained by the nurse is most important to report to the health care provider? a. The patient drinks 1 to 2 cups of coffee daily. b. The patient had a recent acute myocardial infarction. c. The patient has had migraine headaches for 30 years. d. The patient has taken topiramate (Topamax) for 2 months.

b. The patient had a recent acute myocardial infarction. The triptans cause coronary artery vasoconstriction and should be avoided in patients with coronary artery disease. The other information will be reported to the health care provider, but none of it indicates that sumatriptan would be an inappropriate treatment.

The nurse is assessing a patient with osteoarthritis who uses naproxen (Naproxyn) for pain management. Which assessment finding should the nurse recognize as likely to require a change in medication? a. The patient has gained 3 pounds. b. The patient has dark-colored stools. c. The patient's pain affects multiple joints. d. The patient uses capsaicin cream (Zostrix).

b. The patient has dark-colored stools. Dark-colored stools may indicate the patient is experiencing gastrointestinal bleeding caused by the naproxen. The patient's ongoing pain and weight gain will also be reported and may indicate a need for a different treatment and/or counseling about avoiding weight gain, but these are not as large a concern as the possibility of gastrointestinal bleeding. Use of capsaicin cream with oral medications is appropriate.

The son of a dying patient tells the nurse, Mother doesnt really respond any more when I visit. I dont think she knows that I am here. Which response by the nurse is appropriate? a. You may need to cut back your visits for now to avoid overtiring your mother. b. Withdrawal may sometimes be a normal response in the process of dying c. It will be important for you to stimulate your mother as she gets closer to dying. d. Many patients don't really know what is going on around them at the end of life.

b. Withdrawal may sometimes be a normal response in the process of dying

What findings should the nurse expect in the assessment of a young adult with infective endocarditis a. substernal chest pressure. b. a new regurgitant murmur. c. a pruritic rash on the chest.d. involuntary muscle movement.

b. a new regurgitant murmur. Rationale: New regurgitant murmurs occur in IE because vegetations on the valves prevent valve closure. Substernal chest discomfort, rashes, and involuntary muscle movement are clinical manifestations of other cardiac disorders such as angina and rheumatic fever.

A construction worker arrives at an urgent care center with a deep puncture wound from a rusty nail. The patient reports having had a tetanus booster 6 years ago. The nurse will anticipate a. IV infusion of tetanus immune globulin (TIG). b. administration of the tetanus-diphtheria (Td) booster. c. intradermal injection of an immune globulin test dose. d.initiation of the tetanus-diphtheria immunization series.

b. administration of the tetanus-diphtheria (Td) booster.

During the teaching session for a patient who has a new diagnosis of acute leukemia the patient is restless and is looking away, never making eye contact. After teaching about the complications associated with chemotherapy, the patient asks the nurse to repeat all of the information. Based on this assessment, which nursing diagnosis is most appropriate for the patient? a. denial b. anxiety c. acute confusion d. ineffective adherence to treatment

b. anxiety

To assess the patient with pericarditis for evidence of a pericardial friction rub, the nurse should: a. listen for a rumbling, low-pitched, systolic murmur over the left anterior chest. b. auscultate by placing the diaphragm of the stethoscope on the lower left sternal border. c. ask the patient to cough during auscultation to distinguish the sound from a pleural friction rub. d. feel the precordial area with the palm of the hand to detect vibrations with cardiac contraction.

b. auscultate by placing the diaphragm of the stethoscope on the lower left sternal border Rationale: Pericardial friction rubs are best heard with the diaphragm at the lower left sternal border. The nurse should ask the patient to hold his or her breath during auscultation to distinguish the sounds from a pleural friction rub. Friction rubs are not typically low pitched or rumbling and are not confined to systole. Rubs are not assessed by palpation.

A 28-year-old man with von Willebrand disease is admitted to the hospital for minor knee surgery. The nurse will review the coagulation survey to check the a. platelet count. b. bleeding time. c. thrombin time. d. prothrombin time.

b. bleeding time.

A routine complete blood count indicates that an active 80-year-old man may have myelodysplastic syndrome. The nurse will plan to teach the patient about a. blood transfusion b. bone marrow biopsy. c. filgrastim (Neupogen) administrationd. erythropoietin (Epogen) administration.

b. bone marrow biopsy.

A patient with second-degree burns has been receiving hydromorphone through patient-controlled analgesia (PCA) for a week. The patient wakes up frequently during the night complaining of pain. What action by the nurse is most appropriate? a. Administer a dose of morphine every 1 to 2 hours from the PCA machine while the patient is sleeping. b. consult with the HCP about using a different treatment protocol to control the patients pain c. Request that the health care provider order a bolus dose of morphine to be given when the patient awakens with pain.d. Teach the patient to push the button every 10 minutes for an hour before going to sleep, even if the pain is minimal.

b. consult with the HCP about using a different treatment protocol to control the patients pain

When obtaining a health history and physical assessment for a 36-year-old female patient with possible multiple sclerosis (MS), the nurse should a. assess for the presence of chest pain. b. inquire about urinary tract problems. c. inspect the skin for rashes or discoloration. d. ask the patient about any increase in libido

b. inquire about urinary tract problems. Urinary tract problems with incontinence or retention are common symptoms of MS. Chest pain and skin rashes are not symptoms of MS. A decrease in libido is common with MS.

The priority nursing assessment for a patient being admitted with a brainstem infarction is a. pupil reaction. b. respiratory rate. c. reflex reaction time. d. level of consciousness.

b. respiratory rate

To prevent recurrence of uric acid kidney stones, the nurse teaches the patient to avoid eating: a. legumes and dried fruit. b. sardines and liver. c. milk and cheese. d. spinach and chocolate

b. sardines and liver Rationale: Organ meats and fish such as sardines increase purine levels and uric acid. Spinach, chocolate, and tomatoes should be avoided in patients who have oxalate stones. Milk, dairy products, legumes, and dried fruits may increase the incidence of calcium-containing stones.

A 78 year old man with a history of diabetes presents with confusion and temperature of 104 F. A wound on right heel with purulent drainage. After an infusion of 3L of NS, his assessment findings are BP 88/40, HR 110, RR 42 and shallow, CO of 4, and PAWP of 8. This patient's symptoms are most likely indicative of: a. sepsis. b. septic shock. c. multiple organ dysfunction syndrome. d. systemic inflammatory response syndrome.

b. septic shock. Rationale: To meet the diagnostic criteria for sepsis, the patient's temperature must be above 100.9 F, or the core temp must be lower than 97.0 F. Hemodynamic parameters for septic shock include elevated heart rate, and a decreased pulse pressure, systemic vascular resistance, central venous pressure, and pulmonary artery wedge pressure (PAWP). Increased, normal, or detailed cardiac output.

The nurse assessing a patient with newly diagnosed trigeminal neuralgia will ask the patient about a. visual problems caused by ptosis. b. triggers leading to facial discomfort. c. poor appetite caused by loss of taste. d. weakness on the affected side of the face.

b. triggers leading to facial discomfort. The major clinical manifestation of trigeminal neuralgia is severe facial pain that is triggered by cutaneous stimulation of the nerve. Ptosis, loss of taste, and facial weakness are not characteristics of trigeminal neuralgia

Which nursing action can the registered nurse (RN) delegate to unlicensed assistive personnel (UAP) who are assisting with the care of a patient with scleroderma? a.Monitor for difficulty in breathing. b.Document the patient's oral intake. c.Check finger strength and movement. d.Apply capsaicin (Zostrix) cream to hands.

b.Document the patient's oral intake. Monitoring and documenting patients' oral intake is included in UAP education and scope of practice. Assessments for changes in physical status and administration of medications require more education and scope of practice, and should be done by licensed nurses.

Which action should the nurse take before administering gentamicin (Garamycin) to a patient who has acute osteomyelitis? a.Ask the patient about any nausea. b.Review the patient's creatinine level. c.Obtain the patient's oral temperature. d.Change the prescribed wet-to-dry dressing.

b.Review the patient's creatinine level. Gentamicin is nephrotoxic and can cause renal failure. Monitoring the patient's temperature before gentamicin administration is not necessary. Nausea is not a common side effect of IV gentamicin. There is no need to change the dressing before gentamicin administration.

Which action will the nurse include in the plan of care for a 33-year-old patient with a new diagnosis of rheumatoid arthritis? a.Instruct the patient to purchase a soft mattress. b.Suggest that the patient take a nap in the afternoon. c.Teach the patient to use lukewarm water when bathing. d.Suggest exercise with light weights several times daily.

b.Suggest that the patient take a nap in the afternoon. Adequate rest helps decrease the fatigue and pain that are associated with rheumatoid arthritis. Patients are taught to avoid stressing joints, to use warm baths to relieve stiffness, and to use a firm mattress. When stabilized, a therapeutic exercise program is usually developed by a physical therapist to include exercises that improve the flexibility and strength of the affected joints, and the patient's overall endurance.

The nurse is caring for a patient diagnosed with stage I colon cancer. When assessing the need for psychologic support, which question by the nurse will provide the most information? a. "How long ago were you diagnosed with this cancer?" b. "Do you have any concerns about body image changes?" c. "Can you tell me what has been helpful to you in the past when coping with stressful events?" d. "Are you familiar with the stages of emotional adjustment to a diagnosis like cancer of the colon?"

c. "Can you tell me what has been helpful to you in the past when coping with stressful events?"

A patient with heart failure has a new order for captopril 12.5 mg PO. After giving the first dose and teaching the patient about the drug, which statement by the patient indicates that teaching has been effective? a. "I will be sure to take the medication with food." b. "I will need to eat more potassium-rich foods in my diet." c. "I will call for help when I need to get up to use the bathroom." d. "I will expect to feel more short of breath for the next few days."

c. "I will call for help when I need to get up to use the bathroom." rationale: captopril can cause hypotension, especially after the initial dose, so it is important that the patient not get up out of bed without assistance until the nurse has had a chance to evaluate the effect of the first dose.

The nurse teaches a patient with osteoarthritis (OA) of the hip about how to manage the OA. Which patient statement indicates to the nurse a need for additional teaching? a. "A shower in the morning will help relieve stiffness." b. "I can exercise every day to help maintain joint mobility." c. "I will take 1 gram of acetaminophen (Tylenol) every 4 hours." d. "I can use a cane to decrease the pressure and pain in my hip."

c. "I will take 1 gram of acetaminophen (Tylenol) every 4 hours." No more than 4 g of acetaminophen (1 g every 6 hours) should be taken daily to decrease the risk for liver damage. Regular exercise, moist heat, and supportive equipment are recommended for OA management.

A patient with ST-segment elevation in three contiguous electrocardiographic leads is admitted to the emergency department and diagnosed as having an ST-segment-elevation myocardial infarction (STEMI). Which question should the nurse ask to determine whetherthe patient is a candidate for thrombolytic therapy? a. "Do you have any allergies?" b. "Do you take aspirin daily?" c. "What time did your pain begin?" d. "Can you rate the pain on a 0 to 10 scale?

c. "What time did your pain begin?" Rationale: Thrombolytic therapy should be started within 6 hours of the onset of the myocardial infarction, so the time at which the chest pain started is a major determinant of the appropriateness of this treatment. The other information is not a factor in the decision about thrombolytic therapy.

The nurse receives change-of-shift report on the oncology unit. Which patient should the nurse assess first? a. 35-year-old patient who has wet desquamation associated with abdominal radiation b. 42-year-old patient who is sobbing after receiving a new diagnosis of ovarian cancer c. 24-year-old patient who received neck radiation and has blood oozing from the neck d. 56-year-old patient who developed a new pericardial friction rub after chest radiation

c. 24-year-old patient who received neck radiation and has blood oozing from the neck

Immediately after the nurse administers an intracutaneous injection of an allergen on the forearm, a patient complains of itching at the site and of weakness and dizziness. What action should the nurse take first? a. Remind the patient to remain calm. b. Administer subcutaneous epinephrine. c. Apply a tourniquet above the site. d. Rub a local antiinflammatory cream on the site.

c. Apply a tourniquet above the site.

The nurse assesses that a patient receiving epidural morphine has not voided for over 10 hours. What action should the nurse take initially? a. Monitor for withdrawal symptoms. b. Place an indwelling urinary catheter. c. Ask if the patient feels the need to void. d. Document this allergic reaction in the patients chart.

c. Ask if the patient feels the need to void.

A patient admitted with dermal ulcers who has a history of a T3 spinal cord injury tells the nurse, "I have a pounding headache and I feel sick to my stomach." Which action should the nurse take first? a. Check for a fecal impaction. b. Give the prescribed analgesic. c. Assess the blood pressure (BP). d. Notify the health care provider.

c. Assess the blood pressure (BP). The BP should be assessed immediately in a patient with an injury at the T6 level or higher who complains of a headache to determine whether autonomic dysreflexia is occurring. Notification of the patient's health care provider is appropriate after the BP is obtained. Administration of an antiemetic is indicated after autonomic dysreflexia is ruled out as the cause of the nausea. After checking the BP, the nurse may assess for a fecal impaction using lidocaine jelly to prevent further increased BP.

A new mother expresses concern about her baby developing allergies and asks what the health care provider meant by passive immunity. Which example should the nurse use to explain this type of immunity? a. Early immunization b. Bone marrow donation c. Breastfeeding her infant d. Exposure to communicable diseases

c. Breastfeeding her infant

What should the nurse include when teaching older adults at a community recreation center about ways to prevent fractures? a. Tack down scatter rugs on the floor in the home. b. Expect most falls to happen outside the home in the yard. c. Buy shoes that provide good support and are comfortable to wear. d. Get instruction in range-of-motion exercises from a physical therapist.

c. Buy shoes that provide good support and are comfortable to wear Comfortable shoes with good support will help decrease the risk for falls. Scatter rugs should be eliminated, not just tacked down. Activities of daily living provide range-of-motion exercise; these do not need to be taught by a physical therapist. Falls inside the home are responsible for many injuries.

The nurse will anticipate the need to teach a 57-year-old patient who has osteoarthritis (OA) about which medication? a. Adalimumab (Humira) b. Prednisone (Deltasone) c. Capsaicin cream (Zostrix) d. Sulfasalazine (Azulfidine)

c. Capsaicin cream (Zostrix) Capsaicin cream blocks the transmission of pain impulses and is helpful for some patients in treating OA. The other medications would be used for patients with RA.

A 70-year-old female patient with left-sided hemiparesis arrives by ambulance to the emergency department. Which action should the nurse take first? a. Monitor the blood pressure. b. Send the patient for a computed tomography (CT) scan. c. Check the respiratory rate and effort. d. Assess the Glasgow Coma Scale score.

c. Check the respiratory rate and effort.

External-beam radiation is planned for a patient with cervical cancer. What instructions should the nurse give to the patient to prevent complications from the effects of the radiation? a. Test all stools for the presence of blood. b. Maintain a high-residue, high-fiber diet. c. Clean the perianal area carefully after every bowel movement. d. Inspect the mouth and throat daily for the appearance of thrush.

c. Clean the perianal area carefully after every bowel movement.

Diltiazem (Cardizem) is prescribed for a patient with newly diagnosed Prinzmetal's (variant) angina. Which action of diltiazem is accurate for the nurse to include in the teaching plan? a. Reduces heart palpitations. b. Prevents coronary artery plaque. c. Decreases coronary artery spasms. d. Increases contractile force of the heart.

c. Decreases coronary artery spasms. Rationale: Prinzmetal's angina is caused by coronary artery spasm. Calcium channel blockers (e.g., diltiazem, amlodipine [Norvasc]) are a first-line therapy for this type of angina. Lipid-lowering drugs help reduce atherosclerosis (i.e., plaque formation), and β-adrenergic blockers decrease sympathetic stimulation of the heart (i.e., palpitations). Medications or activities that increase myocardial contractility will increase the incidence of angina by increasing oxygen demand.

Which finding in a patient with a spinal cord tumor is most important for the nurse to report to the health care provider? a. Back pain that increases with coughing b. Depression about the diagnosis of a tumor c. Decreasing sensation and ability to move the legs d. Anxiety about scheduled surgery to remove the tumor

c. Decreasing sensation and ability to move the legs Decreasing sensation and leg movement indicates spinal cord compression, an emergency that will require rapid action (such as surgery) to prevent paralysis. The other findings will also require nursing action but are not emergencies.

A patient treated for human immunodeficiency virus (HIV) infection for 6 years has developed fat redistribution to the trunk, with wasting of the arms, legs, and face. What instructions will the nurse give to the patient? a. Review foods that are higher in protein. b. Teach about the benefits of daily exercise. c. Discuss a change in antiretroviral therapy. d. Talk about treatment with antifungal agents.

c. Discuss a change in antiretroviral therapy.

A patient who has chest pain is admitted to the ED and all of the following items are prescribed. Which one should the nurse arrange to be completed first? a. Chest x-ray b. Troponin level c. Electrocardiogram (ECG) d. Insertion of a peripheral IV

c. Electrocardiogram (ECG) Rationale: The priority for the patient is to determine whether an acute myocardial infarction (AMI) is occurring so that the appropriate therapy can begin as quickly as possible. ECG changes occur very rapidly after coronary artery occlusion, and an ECG should be obtained as soon as possible. Troponin levels will increase after about 3 hours. Data from the chest x-ray may impact the patient's care but are not helpful in determining whether the patient is experiencing a myocardial infarction. Peripheral access will be needed but not before the ECG.

A patient who had a total hip replacement had an intraoperative hemorrhage 14 hours ago. Which laboratory result would the nurse expect to find? a. Hematocrit of 46% b. Hemoglobin of 13.8 g/dL c. Elevated reticulocyte count d. Decreased white blood cell (WBC) count

c. Elevated reticulocyte count Hemorrhage causes the release of reticulocytes (immature red blood cells) from the bone marrow into circulation. The hematocrit and hemoglobin levels are normal. The WBC count is not affected by bleeding.

A nurse is caring for a client with endocrine problems. Which lab finding will alert the nurse that aldosterone will be released?

c. Hyponatremia

A 72-yr-old patient with a history of benign prostatic hyperplasia (BPH) is admitted with acute urinary retention and elevated blood urea nitrogen (BUN) and creatinine levels. Which prescribed therapy should the nurse implement first? a. Infuse normal saline at 50 mL/hr. b. Draw a complete blood count. c. Insert urethral catheter. d. Obtain renal ultrasound.

c. Insert urethral catheter. Rationale: The patient's elevation in BUN is most likely associated with hydronephrosis caused by the acute urinary retention, so the insertion of an indwelling catheter is the first action to prevent ongoing postrenal failure for this patient. The other actions are appropriate but should be implemented after the catheter.

A primary healthcare provider diagnoses a client with acute cholecystitis with biliary colic. Which clinical findings should the nurse expect when performing a health history and physical assessment? Select all that apply

c. Intolerance to foods high in fat d. Pain that radiates to the right shoulder

A client admitted to the hospital with a diagnosis of diabetic ketoacidosis (DKA). The initial blood glucose is 950. A continuous IV infusion of short acting insulin is initiated, along with normal saline. The serum glucose level is now decreased to 240. The nurse would next prepare to administer which medication? a. Ampule of 50% dextrose b. NPH insulin subcutaneously c. Intravenous fluids containing dextrose d. Phenytoin (Dilantin) for the prevention of seizures

c. Intravenous fluids containing dextrose

A patient who is unconscious has ineffective cerebral tissue perfusion and cerebral tissue swelling. Which nursing intervention will be included in the plan of care? a. Encourage coughing and deep breathing. b. Position the patient with knees and hips flexed. c. Keep the head of the bed elevated to 30 degrees. d. Cluster nursing interventions to provide rest periods.

c. Keep the head of the bed elevated to 30 degrees. The patient with increased intracranial pressure (ICP) should be maintained in the head-up position to help reduce ICP. Extreme flexion of the hips and knees increases abdominal pressure, which increases ICP. Because the stimulation associated with nursing interventions increases ICP, clustering interventions will progressively elevate ICP. Coughing increases intrathoracic pressure and ICP.

A patient receives 3% NaCl solution for correction of hyponatremia. Which assessment is most important for the nurse to monitor while the patient is receiving this infusion? a. Peripheral pulses b. Peripheral edema c. Lung sounds d. Urinary output

c. Lung sounds Rationale: Hypertonic solutions cause water retention, so the patient should be monitored for symptoms of fluid excess. Crackles in the lungs may indicate the onset of pulmonary edema and are a serious manifestation of fluid excess. Peripheral pulses, peripheral edema, or changes in urine output are also important to monitor when administering hypertonic solutions, but they do not indicate acute respiratory or cardiac decompensation.

A nurse is assessing a newly admitted patient with chronic heart failure who forgot to take prescribed medications. The patient seems confused and short of breath with peripheral edema. Which assessment should the nurse complete first? a. Capillary refill b. Skin turgor c. Mental status d. Heart sounds

c. Mental status Rationale: Increases in extracellular fluid (ECF) can lead to swelling of cells in the central nervous system, initially causing confusion, which may progress to coma or seizures. Although skin turgor, capillary refill, and heart sounds may also be affected by increases in ECF, these are signs that do not have as immediate impact on patient outcomes as cerebral edema.

A patient who is receiving dobutamine for the treatment of acute decompensated heart failure (ADHF) has the following nursing interventions included in the plan of care. Which action will be most appropriate for the registered nurse (RN) to delegate to an experienced licensed practical/vocational nurse (LPN/LVN)? a. Teach the patient the reasons for remaining on bed rest. b. Change the peripheral IV site according to agency policy. c. Monitor the patient's blood pressure and heart rate every hour. d. Titrate the rate to keep the systolic blood pressure >90 mm Hg

c. Monitor the patient's blood pressure and heart rate every hour.

A patient with chronic obstructive pulmonary disease (COPD) has been eating very little and has lost weight. Which intervention would be most appropriate for the nurse to include in the plan of care? a. Assist the patient in choosing foods with high vegetable content. b. Encourage increased intake of whole grains. c. Offer high-calorie protein snacks between meals and at bedtime. d. Increase the patient's menu order of fruits and fruit juices.

c. Offer high-calorie protein snacks between meals and at bedtime. Rationale: Eating small amounts more often (as occurs with snacking) will increase caloric intake by decreasing the fatigue and feelings of fullness associated with large meals. Patients with COPD should rest before meals. Foods that have a lot of texture such as whole grains may take more energy to eat and get absorbed and lead to decreased intake. Although fruits, juices, and minerals are not contraindicated, foods high in protein are a better choice.

A patient recovering from heart surgery develops pericarditis and complains of level 6 (0 to 10 scale) chest pain with deep breathing. Which ordered PRN medication will be the most appropriate for the nurse to give? a. Fentanyl 1 mg IV b. IV morphine sulfate 4 mg c. Oral ibuprofen (Motrin) 600 mg d. Oral acetaminophen (Tylenol) 650 mg

c. Oral ibuprofen (Motrin) 600 mg The pain associated with pericarditis is caused by inflammation, NSAIds (ex: ibuprofen) are most effective.

Which patient should the nurse assess first? a. Patient with urticaria after receiving an IV antibiotic b. Patient who has graft-versus-host disease and severe diarrhea c. Patient who is sneezing after having subcutaneous immunotherapy d. Patient with multiple chemical sensitivities who has muscle stiffness

c. Patient who is sneezing after having subcutaneous immunotherapy

Which hospitalized patient will the nurse assign to the room closest to the nurses' station? a. Patient with Alzheimer's disease who has long-term memory deficit b. Patient with vascular dementia who takes medications for depression c. Patient with new-onset confusion, restlessness, and irritability after surgery d. Patient with dementia who has an abnormal Mini-Mental State Examination

c. Patient with new-onset confusion, restlessness, and irritability after surgery This patient's history and clinical manifestations are consistent with delirium. The patient is at risk for safety problems and should be placed near the nurses' station for ongoing observation. The other patients have chronic symptoms that are consistent with their diagnoses but are not at immediate risk for safety issues.

When caring for a patient with a new right-sided homonymous hemianopsia resulting from a stroke, which intervention should the nurse include in the plan of care? a. Apply an eye patch to the right eye. b. Approach the patient from the right side. c. Place objects needed on the patient's left side. d. Teach the patient that the left visual deficit will resolve.

c. Place objects needed on the patient's left side.

A patient who was involved in a motor vehicle crash has had a tracheostomy placed to allow for continued mechanical ventilation. How should the nurse interpret the following arterial blood gas results: pH 7.48, PaO2 85 mm Hg, PaCO2 32 mm Hg, and HCO3 25 mEq/L? a. Metabolic alkalosis b. Respiratory acidosis c. Respiratory alkalosis d. Metabolic acidosis

c. Respiratory alkalosis Rationale: The pH indicates that the patient has alkalosis and the low PaCO2 indicates a respiratory cause. The other responses are incorrect based on the pH and the normal HCO3.

Which electrocardiographic (ECG) change by a patient with chest pain is most important for the nurse to report rapidly to the health care provider? a. Inverted P wave b. Sinus tachycardia c. ST-segment elevation d. First-degree atrioventricular block

c. ST-segment elevation Rationale: The patient is likely to be experiencing an ST-segment-elevation myocardial infarction (STEMI). Immediate therapy with percutaneous coronary intervention (PCI) or thrombolytic medication is indicated to minimize myocardial damage. The other ECG changes may also suggest a need for therapy, but not as rapidly.

A patient with diabetes who has bacterial pneumonia is being treated with IV gentamicin. What should the nurse monitor for adverse effects of the medication? a. Urine osmolality b. Blood glucose c. Serum creatinine d. Serum potassium

c. Serum creatinine Rationale: When a patient with diabetes, which increases risk for chronic kidney disease (CKD), receives a potentially nephrotoxic medication, it is important to monitor renal function with BUN and creatinine levels. The other laboratory values would not be useful in assessing for the adverse effects of the gentamicin.

The nurse is caring for a patient with left-sided lung cancer. Which finding would be most important for the nurse to report to the health care provider? a. Hematocrit 32% b. Pain with deep inspiration c. Serum sodium 126 mEq/L d. Decreased breath sounds on left side

c. Serum sodium 126 mEq/L

A patient from a long-term care facility is admitted to the hospital with a sacral pressure ulcer. The base of the wound involves subcutaneous tissue. How should the nurse classify this pressure ulcer? a. Stage I b. Stage 2 c. Stage 3 d. Stage 4

c. Stage 3

A patient admitted with possible stroke has been aphasic for 3 hours and has a current blood pressure of 174/94 mm Hg. Which order by the health care provider should the nurse question? a. Keep head of bed elevated at least 30 degrees. b. Infuse normal saline intravenously at 75 mL/hr. c. Start a labetalol drip to keep BP less than 140/90 mm Hg. d. Begin tissue plasminogen activator (tPA) intravenously per protocol.

c. Start a labetalol drip to keep BP less than 140/90 mm Hg. Rationale: Because elevated BP may be a protective response to maintain cerebral perfusion, antihypertensive therapy is recommended only if mean arterial pressure (MAP) is greater than130 mm Hg or systolic pressure is greater than 220 mm Hg. Fluid intake should be 1500 to 2000 mL/day to maintain cerebral blood flow. The head of the bed should be elevated to at least 30 degrees unless the patient has symptoms of poor tissue perfusion. tPA may be administered if the patient meets the other criteria for tPA use.

Which information about a patient with MS indicates that the nurse should consult with the health care provider before giving the prescribed dose of dalfampridine (Ampyra)? a. The patient has relapsing-remitting MS. b. The patient enjoys walking for relaxation. c. The patient has an increased creatinine level. d. The patient complains of pain with neck flexion.

c. The patient has an increased creatinine level.

The nurse is caring for a patient who has a head injury and fractured right arm after being assaulted. Which assessment information requires the most rapid action by the nurse? a. The apical pulse is slightly irregular. b. The patient complains of a headache. c. The patient is more difficult to arouse. d. The blood pressure (BP) increases to 140/62 mm Hg.

c. The patient is more difficult to arouse.

While the nurse is transporting a patient on a stretcher to the radiology department, the patient begins having a tonic-clonic seizure. Which action should the nurse take? a. Insert an oral airway during the seizure to maintain a patent airway. b. Restrain the patient's arms and legs to prevent injury during the seizure. c. Time and observe and record the details of the seizure and postictal state. d. Avoid touching the patient to prevent further nervous system stimulation.

c. Time and observe and record the details of the seizure and postictal state.

When a patient arrives in the emergency department with a facial fracture, which action should the nurse take first? a. Assess for nasal bleeding and pain. b. Apply ice to the face to reduce swelling. c. Use a cervical collar to stabilize the spine. d. Check the patient's alertness and orientation.

c. Use a cervical collar to stabilize the spine. Patients who have facial fractures are at risk for cervical spine injury and should be treated as if they have a cervical spine injury until this is ruled out. The other actions are also necessary, but the most important action is to prevent cervical spine injury.

The nurse is watching the cardiac monitor and notices that the rhythm suddenly changes. There are no P waves, the QRS complexes are wide, and the ventricular rate is regular, but over 100 bpm. The nurse determines that the client is experiencing which of the following dysrhythmias? a. Sinus tachycardia b. Ventricular fibrillation c. Ventricular tachycardia d. Premature ventricular contractions (PVCs)

c. Ventricular tachycardia characterized by the absence of P waves, wide QRS complexes (usually greater than 0.14 second), and a rate between 100 and 250 impulses per minute. The rhythm is usually regular.

The nurse performing a focused assessment of left posterior temporal lobe functions will assess the patient for a. sensation on the left side of the body. b. reasoning and problem-solving ability. c. ability to understand written and oral language. d. voluntary movements on the right side of the body.

c. ability to understand written and oral language. The posterior temporal lobe integrates the visual and auditory input for languagecomprehension. Reasoning and problem solving are functions of the anterior frontal lobe.Sensation on the left side of the body is located in the right postcentral gyrus. Voluntarymovement on the right side is controlled in the left precentral gyrus.

When a brain-injured patient responds to nail bed pressure with internal rotation, adduction, and flexion of the arms, the nurse reports the response as a. flexion withdrawal. b. localization of pain. c. decorticate posturing. d. decerebrate posturing

c. decorticate posturing

A 38-year-old patient has returned home following rehabilitation for a spinal cord injury. The home care nurse notes that the spouse is performing many of the activities that the patient had been managing unassisted during rehabilitation. The most appropriate action by the nurse at this time is to a. remind the patient about the importance of independence in daily activities. b. tell the spouse to stop because the patient is able to perform activities independently. c. develop a plan to increase the pt's independence in consultation with the patient and the spouse d. recognize that it is important for the spouse to be involved in the patient's care and encourage that participation.

c. develop a plan to increase the pt's independence in consultation with the patient and the spouse The best action by the nurse will be to involve all the parties in developing an optimal plan of care. Because family members who will be assisting with the patient's ongoing care need to feel that their input is important, telling the spouse that the patient can perform activities independently is not the best choice. Reminding the patient about the importance of independence may not change the behaviors of the spouse. Supporting the activities of the spouse will lead to ongoing dependency by the patient.

A patient is hospitalized with new onset of Guillain-Barré syndrome. The most essential assessment for the nurse to complete is a. determining level of consciousness. b. checking strength of the extremities. c. observing respiratory rate and effort. d. monitoring the cardiac rate and rhythm.

c. observing respiratory rate and effort. The most serious complication of Guillain-Barré syndrome is respiratory failure, and the nurse should monitor respiratory function continuously. The other assessments will also be included in nursing care, but they are not as important as respiratory assessment.

A patient who has chronic heart failure tells the nurse, "I was fine when I went to bed, but I woke up in the middle of the night feeling like I was suffocating!" The nurse will document this assessment finding as: a. orthopnea. b. pulsus alternans. c. paroxysmal nocturnal dyspnea. d. acute bilateral pleural effusion

c. paroxysmal nocturnal dyspnea. rationale: paroxysmal nocturnal dyspnea is caused by the reabsorption of fluid from dependent body areas when the patient is sleeping and is characterized by waking up suddenly with the feeling of suffocation

A patient who had a C7 spinal cord injury a week ago has a weak cough effort and audible rhonchi. The initial intervention by the nurse should be to a. administer humidified oxygen by mask. b. suction the patient's mouth and nasopharynx. c. push upward on the epigastric area as the patient coughs. d. encourage incentive spirometry every 2 hours during the day.

c. push upward on the epigastric area as the patient coughs. Because the cough effort is poor, the initial action should be to use assisted coughing techniques to improve the ability to mobilize secretions. Administration of oxygen will improve oxygenation, but the data do not indicate hypoxemia. The use of the spirometer may improve respiratory status, but the patient's ability to take deep breaths is limited by the loss of intercostal muscle function. Suctioning may be needed if the patient is unable to expel secretions by coughing but should not be the nurse's first action.

To assess the functions of the trigeminal and facial nerves (CNs V and VII), the nurse should a. check for unilateral eyelid droop. b. shine a light into the patient's pupil. c. touch a cotton wisp strand to the cornea. d. have the patient read a magazine or book.

c. touch a cotton wisp strand to the cornea. The trigeminal and facial nerves are responsible for the corneal reflex. The optic nerve is tested by having the patient read a Snellen chart or a newspaper. Assessment of pupil response to light and ptosis are used to check function of the oculomotor nerve.

During the change of shift report a nurse is told that a patient has an occluded left posterior cerebral artery. The nurse will anticipate that the patient may have a. dysphasia. b. confusion. c. visual deficits. d. poor judgment.

c. visual deficits. Visual disturbances are expected with posterior cerebral artery occlusion. Aphasia occurs with middle cerebral artery involvement. Cognitive deficits and changes in judgment are more typical of anterior cerebral artery occlusion.

Which information will the nurse include when teaching a patient who is scheduled for a radiofrequency catheter ablation for treatment of atrial flutter? a.The procedure will prevent or minimize the risk for sudden cardiac death. b.The procedure will use cold therapy to stop the formation of the flutter waves. c.The procedure will use electrical energy to destroy areas of the conduction system. d.The procedure will stimulate the growth of new conduction pathways between the atria.

c.The procedure will use electrical energy to destroy areas of the conduction system.

A patient is scheduled for a cardiac catheterization with coronary angiography. Before the test, the nurse informs the patient that a. it will be important not to move at all during the procedure. b. monitored anesthesia care will be provided during the procedure. c.a flushed feeling may be noticed when the contrast dye is injected. d. arterial pressure monitoring will be required for 24 hours after the test.

c.a flushed feeling may be noticed when the contrast dye is injected.

The nurse is providing care for a client diagnosed with invasive pancreatic cancer. The client has a permanent biliary drainage tube (T-tube) inserted to provide palliative care. Which action should the nurse take postoperatively?

cleanse the area around the insertion site to prevent skin breakdown

Which patient statement indicates that the nurse's teaching about sublingual nitro has been effective? a. "I can expect nausea as a side effect of nitroglycerin." b. "I should only take nitroglycerin when I have chest pain." c. "Nitroglycerin helps prevent a clot from forming and blocking blood flow to my heart." d. "I will call an ambulance if I have pain after taking 3 nitroglycerin 5 minutes apart

d. "I will call an ambulance if I have pain after taking 3 nitroglycerin 5 minutes apart

The home health nurse cares for an alert and oriented older adult patient with a history of dehydration. Which instructions should the nurse give this patient related to fluid intake? a. "Drink more fluids in the late evening." b. "If you feel confused, you need more fluids." c. "More fluids are needed if you feel thirsty." d. "Increase the fluids if your mouth feels dry."

d. "Increase the fluids if your mouth feels dry." Rationale: An alert older patient will be able to self-assess for signs of oral dryness such as thick oral secretions or dry-appearing mucosa. The thirst mechanism decreases with age and is not an accurate indicator of volume depletion. Many older patients prefer to restrict fluids slightly in the evening to improve sleep quality. The patient will not be likely to notice and act appropriately when changes in level of consciousness occur.

The nurse is caring for a patient with mitral regurgitation. Referring to the figure below, where should the nurse listen to best hear any murmur that the patient has? a. 1 b. 2 c. 3 d. 4 - apex of the heart

d. 4 - apex of the heart Rationale: Sounds from the mitral valve are best heard at the apex of the heart, fifth intercostal space, midclavicular line.

After the emergency department nurse has received a status report on the following patients who have been admitted with head injuries, which patient should the nurse assess first? a. A 20-yr-old patient whose cranial x-ray shows a linear skull fracture b. A 50-yr-old patient who has an initial Glasgow Coma Scale score of 13 c. A 30-yr-old patient who lost consciousness for a few seconds after a fall d. A 50-yr-old patient whose right pupil is 10 mm and unresponsive to light

d. A 50-yr-old patient whose right pupil is 10 mm and unresponsive to light

Which laboratory test will the nurse use to determine whether filgrastim (Neupogen) is effective for a patient with acute lymphocytic leukemia who is receiving chemotherapy? a. Platelet count b. Reticulocyte count c. Total lymphocyte count d. Absolute neutrophil count

d. Absolute neutrophil count

Which prescribed intervention will the nurse implement first for a patient in the emergency department who is experiencing continuous tonic-clonic seizures? a. Give phenytoin (Dilantin) 100 mg IV. b. Monitor level of consciousness (LOC). c. Obtain computed tomography (CT) scan. d. Administer lorazepam (Ativan) 4 mg IV.

d. Administer lorazepam (Ativan) 4 mg IV.

A patient with acute dyspnea is scheduled for a spiral computed tomography (CT) scan. Which information obtained by the nurse is a priority to communicate to the health care provider before the CT? a. Apical pulse of 104 b. Respiratory rate of 30 c. O2 saturation of 90% d. Allergy to shellfish

d. Allergy to shellfish Rationale: Because iodine-based contrast media is used during a spiral CT, the patient may need to have the CT scan without contrast or be premedicated before injection of the contrast media. The increased pulse, low oxygen saturation, and tachypnea all indicate a need for further assessment or intervention but do not indicate a need to modify the CT procedure.

A patient with rheumatoid arthritis has been taking corticosteroids for 11 months. Which nursing action is most likely to detect early signs of infection in this patient? a. Monitor white blood cell count.b. Check the skin for areas of redness. c. Check the temperature every 2 hours. d. Ask about fatigue or feelings of malaise.

d. Ask about fatigue or feelings of malaise.

Which topic will the nurse plan to include in discharge teaching for a patient with heart failure with reduced ejection fraction (HFrEF)? a. Need to begin an aerobic exercise program several times weekly b. Use of salt substitutes to replace table salt when cooking and at the table c. Importance of making an annual appointment with the health care provider d. Benefits and side effects of angiotensin-converting enzyme (ACE) inhibitors

d. Benefits and side effects of angiotensin-converting enzyme (ACE) inhibitors rationale: the core measures for the treatment of heart failure established by The Joint Commission indicate that patients with an ejection fraction (EF) <40% receive an ACE inhibitor to decrease progression of heart failure.

A patient who uses a fentanyl (Duragesic) patch for chronic cancer pain suddenly complains of rapid onset pain at a level 9 (0 to 10 scale) and requests something for pain that will work now. How will the nurse document the type of pain reported by this patient? a. Somatic pain b. Referred pain c. Neuropathic pain d. Breakthrough pain

d. Breakthrough pain

A patient who has just been admitted with pulmonary edema is scheduled to receive the following medications. Which medication should the nurse question before giving. a. Furosemide (Lasix) 60 mg b. Captopril (Capoten) 25 mg c. Digoxin (Lanoxin) 0.125 mg d. Carvedilol (Coreg) 3.125 mg

d. Carvedilol (Coreg) 3.125 mg rationale: Although carbedilol is appropriate for the treatment of chronic heart failure, it is not used for patients with acute decompensated heart failure (ADHF) because of the risk of worsening the heart failure.

A young male patient who is a paraplegic has a stage II sacral pressure ulcer and is being cared for at home by his mother. To prevent further tissue damage, what instructions are most important for the nurse to teach the mother? a. Change the patient's bedding frequently. b. Use a hydrocolloid dressing over the ulcer. c. Record the size and appearance of the ulcer weekly. d. Change the patient's position at least every 2 hours.

d. Change the patient's position at least every 2 hours.

A patient has arrived for a scheduled hemodialysis session. Which nursing action is appropriate for the registered nurse (RN) to delegate to a dialysis technician? a. Teach the patient about fluid restrictions. b. Determine the ultrafiltration rate for the hemodialysis. c. Assess for causes of an increase in predialysis weight. d. Check blood pressure before starting dialysis.

d. Check blood pressure before starting dialysis. Rationale: Dialysis technicians are educated in monitoring for blood pressure. Assessment, adjustment of the appropriate ultrafiltration rate, and patient teaching require the education and scope of practice of an RN.

The nurse is assisting the primary healthcare provider, who is examining the client's skull radiograph. An abnormality in the endocrine gland situated in a depression of the sphenoid bone is suspected. Which hormone release is most probably affected?

d. Corticotropin

A patient admitted to the coronary care unit (CCU) with an ST-segment-elevation myocardial infarction (STEMI) is restless and anxious. The blood pressure is 86/40 mm Hg, and heart rate is 132 beats/min. Based on this information, which nursing diagnosis is a priority for the patient? a. Acute pain related to myocardial infarction b. Anxiety related to perceived threat of death c. Stress overload related to acute change in health d. Decreased cardiac output related to cardiogenic shock

d. Decreased cardiac output related to cardiogenic shock

Which medication taken by a patient with restless legs syndrome should the nurse discuss with the patient? a. Ibuprofen b. Multivitamin c. Acetaminophen d. Diphenhydramine

d. Diphenhydramine

A patient with pancytopenia has a bone marrow aspiration from the left posterior iliac crest. Which action would be important for the nurse to take after the procedure? a. Elevate the head of the bed to 45 degrees. b. Apply a sterile 2-inch gauze dressing to the site. c. Use a half-inch sterile gauze to pack the wound. d. Have the patient lie on the left side for 1 hour.

d. Have the patient lie on the left side for 1 hour.

A patient had a cystectomy with an ileal conduit yesterday. Which new assessment data is most important for the nurse to communicate to the health care provider? a. Cloudy appearing urine b. Continuous stoma drainage c. Hypoactive bowel sounds d. Heart rate 102 beats/min

d. Heart rate 102 beats/min Rationale: Tachycardia may indicate infection, hemorrhage, or hypovolemia, which are all serious complications of this surgery. The urine from an ileal conduit normally contains mucus and is cloudy. Hypoactive bowel sounds are expected after bowel surgery. Continuous drainage of urine from the stoma is normal.

When assessing a newly admitted patient, the nurse notes pallor of the skin and nail beds. The nurse should ensure that which laboratory test has been ordered? a. Platelet count b. Neutrophil count c. White blood cell count d. Hemoglobin (Hgb) level

d. Hemoglobin (Hgb) level

Following successful treatment of Hodgkin's lymphoma for a 55-yr-old woman, which topic will the nurse include in patient teaching? a. Potential impact of chemotherapy treatment on fertility b. Application of soothing lotions to treat residual pruritus c. Use of maintenance chemotherapy to maintain remission d. Need for follow-up appointments to screen for malignancy

d. Need for follow-up appointments to screen for malignancy

A patient with left-sided weakness that started 60 minutes earlier is admitted to the emergency department and diagnostic tests are ordered. Which test should be done first? a. Complete blood count (CBC) b. Chest radiograph (Chest x-ray) c. 12-Lead electrocardiogram (ECG) d. Noncontrast computed tomography (CT)

d. Noncontrast computed tomography (CT) Rapid screening with a noncontrast CT scan is needed before administration of tissue plasminogen activator (tPA), which must be given within 4.5 hours of the onset of clinical manifestations of the stroke. The sooner the tPA is given, the less brain injury. The other diagnostic tests give information about possible causes of the stroke and do not need to be completed as urgently as the CT scan.

The nurse is caring for a 70-yr-old patient who uses hydrochlorothiazide and enalapril (Norvasc) but whose self-monitored blood pressure (BP) continues to be elevated. Which patient information may indicate a need for a change? a. Patient takes a daily multivitamin tablet. b. Patient checks BP daily just after getting up. c. Patient drinks wine three to four times a week. d. Patient uses ibuprofen (Motrin) treat osteoarthritis.

d. Patient uses ibuprofen (Motrin) treat osteoarthritis. Rationale: Because use of nonsteroidal antiinflammatory drugs (NSAIDs) can prevent adequate BP control, the patient may need to avoid the use of ibuprofen. A multivitamin tablet will help supply vitamin D, which may help lower BP. BP decreases while sleeping, so self-monitoring early in the morning will result in obtaining pressures that are at their lowest. The patient's alcohol intake is not excessive.

After change-of-shift report on the Alzheimer's disease/dementia unit, which patient will the nurse assess first? a. Patient who has not had a bowel movement for 5 days b. Patient who has a stage II pressure ulcer on the coccyx c. Patient who is refusing to take the prescribed medications d. Patient who developed a new cough after eating breakfast

d. Patient who developed a new cough after eating breakfast A new cough after a meal in a patient with dementia suggests possible aspiration and the patient should be assessed immediately. The other patients also require assessment and intervention, but not as urgently as a patient with possible aspiration or pneumonia

Which action should the nurse take to prepare a patient with a pleural effusion for a thoracentesis? a. Obtain a collection device to hold 3 liters of pleural fluid. b. Remind the patient not to eat or drink 6 hours. c. Start a peripheral IV line to administer sedation. d. Position the patient sitting up on the side of the bed.

d. Position the patient sitting up on the side of the bed. Rationale: When the patient is sitting up, fluid accumulates in the pleural space at the lung bases and can more easily be located and removed. The patient does not usually require sedation for the procedure, and there are no restrictions on oral intake because the patient is not sedated or unconscious. Usually only 1000 to 1200 mL of pleural fluid is removed at one time. Rapid removal of a large volume can result in hypotension, hypoxemia, or pulmonary edema.

When a patient with hypertension who has a new prescription for atenolol (Tenormin) returns to the health clinic after 2 weeks for a follow-up visit, the BP is unchanged from the previous visit. Which action should the nurse take first? a. Provide information about the use of multiple drugs to treat hypertension. b. Teach the patient about the reasons for a possible change in drug therapy. c. Remind the patient that lifestyle changes also are important in BP control. d. Question the patient about whether the medication is actually being taken.

d. Question the patient about whether the medication is actually being taken. Rationale: Since noncompliance with antihypertensive therapy is common, the nurse's initial action should be to determine whether the patient is taking the atenolol as prescribed. The other actions also may be implemented, but these would be done after assessing patient compliance with the prescribed therapy.

A patient is admitted to the emergency department with severe fatigue and confusion. Laboratory studies are done. Which laboratory value will require the most immediate action by the nurse? a. Arterial blood pH is 7.32. b. Arterial oxygen saturation is 91%. c. Serum potassium is 5.1 mEq/L. d. Serum calcium is 18 mg/dL.

d. Serum calcium is 18 mg/dL Rationale: The serum calcium is well above the normal level and puts the patient at risk for cardiac dysrhythmias. The nurse should start cardiac monitoring and notify the health care provider. The potassium, oxygen saturation, and pH are also abnormal, and the nurse should notify the health care provider about these values as well, but they are not immediately life threatening.

The nurse is caring for a patient who was admitted the previous day with a basilar skull fracture after a motor vehicle crash. Which assessment finding is most important to report to the health care provider? a. Complaint of severe headache b. Large contusion behind left ear c. Bilateral periorbital ecchymosis d. Temperature of 101.4° F (38.6° C

d. Temperature of 101.4° F (38.6° C Patients who have basilar skull fractures are at risk for meningitis, so the elevated temperature should be reported to the health care provider. The other findings are typical of a patient with a basilar skull fracture.

A patient in the emergency department with sudden-onset right-sided weakness is diagnosed with an intracerebral hemorrhage. Which information about the patient is most important to communicate to the health care provider? a. The patient's speech is difficult to understand. b. The patient's blood pressure is 144/90 mm Hg. c. The patient takes a diuretic because of a history of hypertension. d. The patient has atrial fibrillation and takes warfarin (Coumadin).

d. The patient has atrial fibrillation and takes warfarin (Coumadin).

When caring for a patient after cystoscopy, what should the nurse include in the plan of care? a. The patient learns to request narcotics for pain. b. The patient restricts activity to bed rest for 4 to 6 hours. c. The patient remains NPO for 8 hours to prevent vomiting. d. The patient understands to expect blood-tinged urine.

d. The patient understands to expect blood-tinged urine. Rationale: Pink-tinged urine and urinary frequency are expected after cystoscopy. Burning on urination is common, but pain that requires opioids for relief is not expected. A good fluid intake is encouraged after this procedure. Bed rest is not required after cystoscopy.

A patient with human immunodeficiency virus (HIV) infection has developed Mycobacterium avium complex infection. Which outcome would be appropriate for the nurse to include in the plan of care? a. The patient will be free from injury. b. The patient will receive immunizations. c. The patient will have adequate oxygenation. d. The patient will maintain intact perineal skin.

d. The patient will maintain intact perineal skin.

The nurse is obtaining a health history from a 24-year-old patient with hypertrophic cardiomyopathy (HC). Which information obtained by the nurse is most important? a. The patient has a history of a recent upper respiratory infection. b. The patient has a family history of coronary artery disease (CAD). c. The patient reports using cocaine a "couple of times" as a teenager. d. The patient's 29-year-old brother died from a sudden cardiac arrest.

d. The patient's 29-year-old brother died from a sudden cardiac arrest. Rationale: About half of all cases of HC have a genetic basis, and it is the most common cause of sudden cardiac death in otherwise healthy young people. The information about the patient's brother will be helpful in planning care (such as an automatic implantable cardioverter-defibrillator [AICD

When assessing a 53-year-old patient with bacterial meningitis, the nurse obtains the following data. Which finding should be reported immediately to the health care provider? a. The patient exhibits nuchal rigidity. b. The patient has a positive Kernig's sign. c. The patient's temperature is 101° F (38.3° C). d. The patient's blood pressure is 88/42 mm Hg.

d. The patient's blood pressure is 88/42 mm Hg. Rationale: Shock is a serious complication of meningitis, and the patient's low blood pressure indicates the need for interventions such as fluids or vasopressors. Nuchal rigidity and a positive Kernig's sign are expected with bacterial meningitis. The nurse should intervene to lower the temperature, but this is not as life threatening as the hypotension.

The nurse is caring for a mechanically ventilated patient with a cuffed tracheostomy tube. Which action by the nurse would determine if the cuff has been properly inflated? a. Insert the decannulation plug before removing the nonfenestrated inner cannula. b. Suction the patient through a fenestrated inner cannula to clear secretions. c. Review the health record for the prescribed cuff pressure. d. Use a hand-held manometer to measure cuff pressure.

d. Use a hand-held manometer to measure cuff pressure. Rationale: Measurement of cuff pressure using a manometer to ensure that cuff pressure is 20 mm Hg or lower will avoid compression of the tracheal wall and capillaries. Never insert the decannulation plug in a tracheostomy tube until the cuff is deflated and the nonfenestrated inner cannula is removed. Otherwise, the patient's airway is occluded. A health care provider's order is not required to determine safe cuff pressure. A nonfenestrated inner cannula must be used to suction a patient to prevent tracheal damage occurring from the suction catheter passing through the fenestrated openings.

Which action will be included in the plan of care when the nurse is caring for a patient who is receiving nicardipine (Cardene) to treat a hypertensive emergency? a. Organize nursing activities so that the patient has undisturbed sleep for 8 hours at night. b. Keep the patient NPO to prevent aspiration caused by nausea and possible vomiting. c. Assist the patient up in the chair for meals to avoid complications associated with immobility. d. Use an automated noninvasive blood pressure machine to obtain frequent measurements.

d. Use an automated noninvasive blood pressure machine to obtain frequent measurements. Rationale: Frequent monitoring of BP is needed when the patient is receiving rapid-acting IV antihypertensive medications. This can be most easily accomplished with an automated BP machine or arterial line. The patient will require frequent assessments, so allowing 8 hours of undisturbed sleep is not reasonable. When patients are receiving IV vasodilators, bed rest is maintained to prevent decreased cerebral perfusion and fainting. There is no indication that this patient is nauseated or at risk for aspiration, so an NPO status is unnecessary.

The nurse develops a teaching plan to help increase activity tolerance at home for an older adult with severe chronic obstructive pulmonary disease (COPD). Which instructions would be appropriate for the nurse to include in the plan of care? a. Stop exercising when you feel short of breath. b. Walk until pulse rate exceeds 130 beats/min. c. Limit exercise to activities of daily living (ADLs). d. Walk 15 to 20 minutes a day at least 3 times/wk.

d. Walk 15 to 20 minutes a day at least 3 times/wk. Rationale: Encourage the patient to walk 15 to 20 minutes a day at least three times a week with gradual increases. Shortness of breath is normal with exercise and not an indication that the patient should stop. Limiting exercise to ADLs will not improve the patient's exercise tolerance. A 70-yr-old patient should have a pulse rate of 120 beats/min or less with exercise (80% of the maximal heart rate of 150 beats/min).

After receiving change-of-shift report about the following four patients on the cardiac care unit, which patient should the nurse assess first? a. A 39-yr-old patient with pericarditis who is complaining of sharp, stabbing chest pain b. A 56-yr-old patient with variant angina who is scheduled to receive nifedipine (Procardia) c. A 65-yr-old patient who had a myocardial infarction (MI) 4 days ago and is anxious about today's planned discharge d. a 59-yr-old pt with unstable angina who has just returned after a percutaneous coronary intervention (PCI)

d. a 59-yr-old pt with unstable angina who has just returned after a percutaneous coronary intervention (PCI) Rationale: After PCI, the patient is at risk for hemorrhage from the arterial access site. The nurse should assess the patient's blood pressure, pulses, and the access site immediately. The other patients should also be assessed as quickly as possible, but assessment of this patient has the highest priority.

Older adults who become ill are more likely than younger adults to: a. complain about the symptoms of their problems b. refuse to carry out lifestyle changes to promote recovery c. seek medical attention because of limitations on their lifestyle d. alter their daily living activities to accommodate new symptoms

d. alter their daily living activities to accommodate new symptoms

While caring for a 23-year-old patient with mitral valve prolapse (MVP) without valvular regurgitation, the nurse determines that discharge teaching has been effective when the patient states that it will be necessary to: a. take antibiotics before any dental appointments. b. limit physical activity to avoid stressing the heart. c. take an aspirin a day to prevent clots from forming on the valve. d. avoid use of over-the-counter (OTC) medications that contain stimulant drugs.

d. avoid use of over-the-counter (OTC) medications that contain stimulant drugs. Use of stimulant medications should be avoided by patients with MVP because these may exacerbate symptoms. Daily aspirin and restricted physical activity are not needed by patients with mild MVP. Antibiotic prophylaxis is needed for patients with MVP with regurgitation but will not be necessary for this patient

A patient has a normal cardiac rhythm and a heart rate of 72 beats/minute. The nurse determines that the P-R interval is 0.24 seconds. The most appropriate intervention by the nurse would be to a. notify the health care provider immediately. b. give atropine per agency dysrhythmia protocol. c. prepare the patient for temporary pacemaker insertion. d. document the finding and continue to monitor the patient.

d. document the finding and continue to monitor the patient.

A 74-yr-old patient has just arrived in the emergency department. After assessment reveals a pulse deficit of 46 beats, the nurse will anticipate that the patient may require a. emergent cardioversion. b. a cardiac catheterization. c. hourly blood pressure (BP) checks. d. electrocardiographic (ECG) monitoring.

d. electrocardiographic (ECG) monitoring. Rationale: Pulse deficit is a difference between simultaneously obtained apical and radial pulses. It indicates that there may be a cardiac dysrhythmia that would best be detected with ECG monitoring. Frequent BP monitoring, cardiac catheterization, and emergent cardioversion are used for diagnosis and/or treatment of cardiovascular disorders but would not be as helpful in determining the immediate reason for the pulse deficit.

A patient with suspected meningitis is scheduled for a lumbar puncture. Before the procedure, the nurse will plan to a. enforce NPO status for 4 hours. b. transfer the patient to radiology. c. administer a sedative medication. d. help the patient to a lateral position.

d. help the patient to a lateral position. For a lumbar puncture, the patient lies in the lateral recumbent position. The procedure does not usually require a sedative, is done in the patient room, and has no risk for aspiration.

A patient at the clinic says, "I always walk after dinner, but lately my leg cramps and hurts after just a few minutes of starting. The pain goes away after I stop walking, though." The nurse should a. look for the presence of tortuous veins bilaterally on the legs. b. ask about any skin color changes that occur in response to cold. c. assess for unilateral swelling, redness, and tenderness of either leg. d. palpate for the presence of dorsalis pedis and posterior tibial pulses.

d. palpate for the presence of dorsalis pedis and posterior tibial pulses. The nurse should assess for other clinical manifestations of peripheral arterial disease in a patient who describes intermittent claudication. Changes in skin color that occur in response to cold are consistent with Raynaud's phenomenon. Tortuous veins on the legs suggest venous insufficiency. Unilateral leg swelling, redness, and tenderness indicate venous thromboembolism.

Nadolol (Corgard) is prescribed for a patient with chronic stable angina and left ventricular dysfunction. To determine whether the drug is effective, the nurse will monitor for: a. decreased blood pressure and heart rate. b. fewer complaints of having cold hands and feet. c. improvement in the strength of the distal pulses. d. participation in daily activities without chest pain.

d. participation in daily activities without chest pain.

The nurse has identified a nursing diagnosis of acute pain related to inflammatory process for a patient with acute pericarditis. The priority intervention by the nurse for this problem is to a. teach the patient to take deep, slow breaths to control the pain b. force fluids to 3000 mL/day to decrease fever and inflammation. c. remind the patient to request opioid pain medication every 4 hours. d. place the patient in Fowler's position, leaning forward on the overbed table.

d. place the patient in Fowler's position, leaning forward on the overbed table. Rationale: Sitting upright and leaning forward frequently will decrease the pain associated with pericarditis. Forcing fluids will not decrease the inflammation or pain. Taking deep breaths will tend to increase pericardial pain. Opioids are not very effective at controlling pain caused by acute inflammatory conditions and are usually ordered PRN. The patient would receive scheduled doses of a nonsteroidal antiinflammatory drug (NSAID).

The nurse identifies a patient with type 1 diabetes and a history of herpes simplex infection as being at risk for Bell's palsy. Which information should the nurse include in teaching the patient? a."You may be able to prevent Bell's palsy by doing facial exercises regularly." b."Prophylactic treatment of herpes with antiviral agents prevents Bell's palsy." c."Medications to treat Bell's palsy work only if started before paralysis onset." d."Call the doctor if you experience pain or develop herpes lesions near the ear."

d."Call the doctor if you experience pain or develop herpes lesions near the ear." Pain or herpes lesions near the ear may indicate the onset of Bell's palsy and rapid corticosteroid treatment may reduce the duration of Bell's palsy symptoms. Antiviral therapy for herpes simplex does not reduce the risk for Bell's palsy. Corticosteroid therapy will be most effective in reducing symptoms if started before paralysis is complete but will still be somewhat effective when started later. Facial exercises do not prevent Bell's palsy.

The nurse has received change-of-shift report about the following patients on the progressive care unit. Which patient should the nurse see first? a. A patient who is in a sinus rhythm, rate 98, after having electrical cardioversion 2 hours ago b.A patient with new onset atrial fibrillation, rate 88, who has a first dose of warfarin (Coumadin) due c.A patient with second-degree atrioventricular (AV) block, type 1, rate 60, who is dizzy when ambulating d.A patient whose implantable cardioverter-defibrillator (ICD) fired two times today who has a dose of amiodarone (Cordarone) due

d.A patient whose implantable cardioverter-defibrillator (ICD) fired two times today who has a dose of amiodarone (Cordarone) due Rationale: The frequent firing of the ICD indicates that the patient's ventricles are very irritable and the priority is to assess the patient and give the amiodarone. The other patients can be seen after the amiodarone is given.

The nurse is reviewing lab test results for a client admitted to the burn unit after an explosion that occurred at his work site. The client has a severe burn injury that covers 35% of the total body surface area (TBSA). You're most likely to note which findings on the lab results?

hematocrit 60% Rationale: excessive burns of more than 25% TBSA result in generalized body edema in both burned and unburned tissue, and decrease in the intravascular blood volume (which is because of the shift of fluid)


Ensembles d'études connexes

ch. 13 Medical Math, ch. 7:4 Skeletal System, ch. 7 Anatomy and Physiology

View Set

Ap Euro Semester 1 Multiple Choice

View Set

Kyle and Carman: Essentials of Pediatric Nursing

View Set

Phase II: Quiz 4 (100 Questions)

View Set

bio study question 13, 14, bonus

View Set

Astronomy Chapter 23 Process of Science

View Set

ND Life - Federal Tax Considerations for Life Insurance

View Set